Você está na página 1de 50

Ana Larissa Adorno Marciotto Oliveira

Barbara Orfanó

English Semantics

REIMPRESSÃO

Montes Claros/MG - Julho/2015


Copyright ©: Universidade Estadual de Montes Claros
UNIVERSIDADE ESTADUAL DE MONTES CLAROS - UNIMONTES

REITOR Cláudia Regina Santos de Almeida


João dos Reis Canela Fernando Guilherme Veloso Queiroz
Luciana Mendes Oliveira
VICE-REITORA Maria Ângela Lopes Dumont Macedo
Antônio Alvimar Souza Maria Aparecida Pereira Queiroz
Maria Nadurce da Silva
DIRETOR DE DOCUMENTAÇÃO E INFORMAÇÕES Mariléia de Souza
Jânio Marques Dias Priscila Caires Santana Afonso
Zilmar Santos Cardoso
EDITORA UNIMONTES
Conselho Consultivo REVISÃO DE LÍNGUA PORTUGUESA
Adelica Aparecida Xavier Carla Roselma Athayde Moraes
Alfredo Maurício Batista de Paula Waneuza Soares Eulálio
Antônio Dimas Cardoso
Carlos Renato Theóphilo, REVISÃO TÉCNICA
Casimiro Marques Balsa Gisléia de Cássia Oliveira
Elton Dias Xavier Káthia Silva Gomes
José Geraldo de Freitas Drumond Viviane Margareth Chaves Pereira Reis
Laurindo Mékie Pereira
Otávio Soares Dulci DESENVOLVIMENTO DE TECNOLOGIAS EDUCACIONAIS
Marcos Esdras Leite Andréia Santos Dias
Marcos Flávio Silveira Vasconcelos Dângelo Camilla Maria Silva Rodrigues
Regina de Cássia Ferreira Ribeiro Sanzio Mendonça Henriques
Wendell Brito Mineiro
CONSELHO EDITORIAL
Ângela Cristina Borges CONTROLE DE PRODUÇÃO DE CONTEÚDO
Arlete Ribeiro Nepomuceno Camila Pereira Guimarães
Betânia Maria Araújo Passos Joeli Teixeira Antunes
Carmen Alberta Katayama de Gasperazzo Magda Lima de Oliveira
César Henrique de Queiroz Porto Zilmar Santos Cardoso

Catalogação: Biblioteca Central Professor Antônio Jorge - Unimontes


Ficha Catalográfica:

2015
Proibida a reprodução total ou parcial. Os infratores serão processados na forma da lei.

EDITORA UNIMONTES
Campus Universitário Professor Darcy Ribeiro, s/n - Vila Mauricéia - Montes Claros (MG) - Caixa Postal: 126 - CEP: 39.401-089
Correio eletrônico: editora@unimontes.br - Telefone: (38) 3229-8214
Ministro da Educação Diretora do Centro de Ciências Biológicas da Saúde - CCBS/
Renato Janine Ribeiro Unimontes
Maria das Mercês Borem Correa Machado
Presidente Geral da CAPES
Jorge Almeida Guimarães Diretora do Centro de Ciências Humanas - CCH/Unimontes
Mariléia de Souza
Diretor de Educação a Distância da CAPES
Jean Marc Georges Mutzig Diretor do Centro de Ciências Sociais Aplicadas - CCSA/Unimontes
Paulo Cesar Mendes Barbosa
Governador do Estado de Minas Gerais
Fernando Damata Pimentel Chefe do Departamento de Comunicação e Letras/Unimontes
Maria Generosa Ferreira Souto
Secretário de Estado de Ciência, tecnologia e Ensino Superior
Vicente Gamarano Chefe do Departamento de Educação/Unimontes
Maria Cristina Freire Barbosa
Reitor da Universidade Estadual de Montes Claros - Unimontes
João dos Reis Canela Chefe do Departamento de Educação Física/Unimontes
Rogério Othon Teixeira Alves
Vice-Reitor da Universidade Estadual de Montes Claros -
Unimontes Chefe do Departamento de Filosofia/Unimontes
Antônio Alvimar Souza Alex Fabiano Correia Jardim

Pró-Reitor de Ensino/Unimontes Chefe do Departamento de Geociências/Unimontes


João Felício Rodrigues Neto Anete Marília Pereira

Diretor do Centro de Educação a Distância/Unimontes Chefe do Departamento de História/Unimontes


Fernando Guilherme Veloso Queiroz Claudia de Jesus Maia

Coordenadora da UAB/Unimontes Chefe do Departamento de Estágios e Práticas Escolares


Maria Ângela Lopes Dumont Macedo Cléa Márcia Pereira Câmara

Coordenadora Adjunta da UAB/Unimontes Chefe do Departamento de Métodos e técnicas Educacionais


Betânia Maria Araújo Passos Káthia Silva Gomes

Chefe do Departamento de Política e Ciências Sociais/Unimontes


Carlos Caixeta de Queiroz
Autoras
Ana Larissa Adorno Marciotto Oliveira
She has a PhD in Applied Linguistics and currently works at UFMG, where she teaches
English and Applied Linguistics.

Barbara Orfanó
has a PhD in Applied Linguistcs and currently works at UFSJ, where she teaches
English and Applied Linguistics.
Sumário
Presentation . . . . . . . . . . . . . . . . . . . . . . . . . . . . . . . . . . . . . . . . . . . . . . . . 9

Unit 1 . . . . . . . . . . . . . . . . . . . . . . . . . . . . . . . . . . . . . . . . . . . . . . . . . . . . . . 11
Basic Concept . . . . . . . . . . . . . . . . . . . . . . . . . . . . . . . . . . . . . . . . . . . . . . . . . . . . . . . . . . . . . . . . . . . . . . 11

1.1 Introduction . . . . . . . . . . . . . . . . . . . . . . . . . . . . . . . . . . . . . . . . . . . . . . . . . . . . . . . . . . . . . . . . . . . 11

1.2 Sentences, Utterances and Propositions . . . . . . . . . . . . . . . . . . . . . . . . . . . . . . . . . . . . . . . . . 11

1.3 Meaning and Dictionary . . . . . . . . . . . . . . . . . . . . . . . . . . . . . . . . . . . . . . . . . . . . . . . . . . . . . . . . 12

1.4 English Language Dictionaries . . . . . . . . . . . . . . . . . . . . . . . . . . . . . . . . . . . . . . . . . . . . . . . . . . 13

1.5 Lexical Semantics . . . . . . . . . . . . . . . . . . . . . . . . . . . . . . . . . . . . . . . . . . . . . . . . . . . . . . . . . . . . . . . 14

1.6 How Semantics Operates: the Two Levels . . . . . . . . . . . . . . . . . . . . . . . . . . . . . . . . . . . . . . . . 15

1.7 Sense Relations . . . . . . . . . . . . . . . . . . . . . . . . . . . . . . . . . . . . . . . . . . . . . . . . . . . . . . . . . . . . . . . . . 15

1.8 Semantic Field . . . . . . . . . . . . . . . . . . . . . . . . . . . . . . . . . . . . . . . . . . . . . . . . . . . . . . . . . . . . . . . . . . 17

1.9 Metaphor . . . . . . . . . . . . . . . . . . . . . . . . . . . . . . . . . . . . . . . . . . . . . . . . . . . . . . . . . . . . . . . . . . . . . . 17

1.10 Language Variation and the Issue of Correctness . . . . . . . . . . . . . . . . . . . . . . . . . . . . . . . .18

References . . . . . . . . . . . . . . . . . . . . . . . . . . . . . . . . . . . . . . . . . . . . . . . . . . . . . . . . . . . . . . . . . . . . . . . . . 18

Unit 2 . . . . . . . . . . . . . . . . . . . . . . . . . . . . . . . . . . . . . . . . . . . . . . . . . . . . . . 19
Reference and Sense . . . . . . . . . . . . . . . . . . . . . . . . . . . . . . . . . . . . . . . . . . . . . . . . . . . . . . . . . . . . . . . 19

2.1 Introduction . . . . . . . . . . . . . . . . . . . . . . . . . . . . . . . . . . . . . . . . . . . . . . . . . . . . . . . . . . . . . . . . . . . 19

2.2 Cataphoric and Anaphoric References . . . . . . . . . . . . . . . . . . . . . . . . . . . . . . . . . . . . . . . . . . . 20

2.3 Anaphoric Reference . . . . . . . . . . . . . . . . . . . . . . . . . . . . . . . . . . . . . . . . . . . . . . . . . . . . . . . . . . . 20

2.4 Exophoric Reference . . . . . . . . . . . . . . . . . . . . . . . . . . . . . . . . . . . . . . . . . . . . . . . . . . . . . . . . . . . . 20

2.5 Reference and Sense in Descriptive Meaning . . . . . . . . . . . . . . . . . . . . . . . . . . . . . . . . . . . . 21

References . . . . . . . . . . . . . . . . . . . . . . . . . . . . . . . . . . . . . . . . . . . . . . . . . . . . . . . . . . . . . . . . . . . . . . . . . 21

Unit 3 . . . . . . . . . . . . . . . . . . . . . . . . . . . . . . . . . . . . . . . . . . . . . . . . . . . . . . 23
Word Meaning in Dictionaries . . . . . . . . . . . . . . . . . . . . . . . . . . . . . . . . . . . . . . . . . . . . . . . . . . . . . . 23

3.1 Introduction . . . . . . . . . . . . . . . . . . . . . . . . . . . . . . . . . . . . . . . . . . . . . . . . . . . . . . . . . . . . . . . . . . . 23

3.2 The Organization of Dictionaries . . . . . . . . . . . . . . . . . . . . . . . . . . . . . . . . . . . . . . . . . . . . . . . . 23

3.3 Encyclopedia Versus Dictionary Meaning . . . . . . . . . . . . . . . . . . . . . . . . . . . . . . . . . . . . . . . 24

3.4 Sense Relations: Representing Semantic Information . . . . . . . . . . . . . . . . . . . . . . . . . . . . 24


3.5 Properties of Predicates . . . . . . . . . . . . . . . . . . . . . . . . . . . . . . . . . . . . . . . . . . . . . . . . . . . . . . . . 25

3.6 Introduction to Derivation . . . . . . . . . . . . . . . . . . . . . . . . . . . . . . . . . . . . . . . . . . . . . . . . . . . . . . 26

3.7 Compound Nouns . . . . . . . . . . . . . . . . . . . . . . . . . . . . . . . . . . . . . . . . . . . . . . . . . . . . . . . . . . . . . . 27

3.8 Types of Derivation . . . . . . . . . . . . . . . . . . . . . . . . . . . . . . . . . . . . . . . . . . . . . . . . . . . . . . . . . . . . . 27

3.9 Participant Roles . . . . . . . . . . . . . . . . . . . . . . . . . . . . . . . . . . . . . . . . . . . . . . . . . . . . . . . . . . . . . . . .28

3.10 Summing up This Unit . . . . . . . . . . . . . . . . . . . . . . . . . . . . . . . . . . . . . . . . . . . . . . . . . . . . . . . . . 29

References . . . . . . . . . . . . . . . . . . . . . . . . . . . . . . . . . . . . . . . . . . . . . . . . . . . . . . . . . . . . . . . . . . . . . . . . 29

Unit 4 . . . . . . . . . . . . . . . . . . . . . . . . . . . . . . . . . . . . . . . . . . . . . . . . . . . . . . 31
Interpersonal and Non-Literal Meaning . . . . . . . . . . . . . . . . . . . . . . . . . . . . . . . . . . . . . . . . . . . . . 31

4.1 Introduction . . . . . . . . . . . . . . . . . . . . . . . . . . . . . . . . . . . . . . . . . . . . . . . . . . . . . . . . . . . . . . . . . . . . 31

4.2 Speech Act . . . . . . . . . . . . . . . . . . . . . . . . . . . . . . . . . . . . . . . . . . . . . . . . . . . . . . . . . . . . . . . . . . . . . 31

4.3 Perlocutions and Illocutions . . . . . . . . . . . . . . . . . . . . . . . . . . . . . . . . . . . . . . . . . . . . . . . . . . . . 31

4.4 Felicity Conditions . . . . . . . . . . . . . . . . . . . . . . . . . . . . . . . . . . . . . . . . . . . . . . . . . . . . . . . . . . . . . . 32

4.5 Direct and Indirect Form . . . . . . . . . . . . . . . . . . . . . . . . . . . . . . . . . . . . . . . . . . . . . . . . . . . . . . . . 33

4.6 Inference . . . . . . . . . . . . . . . . . . . . . . . . . . . . . . . . . . . . . . . . . . . . . . . . . . . . . . . . . . . . . . . . . . . . . . . 33

4.7 Entailments . . . . . . . . . . . . . . . . . . . . . . . . . . . . . . . . . . . . . . . . . . . . . . . . . . . . . . . . . . . . . . . . . . . . 33

4.8 Conversational Implicature . . . . . . . . . . . . . . . . . . . . . . . . . . . . . . . . . . . . . . . . . . . . . . . . . . . . . 34

4.9 Non-literal Meaning: Idioms, Metaphor, and Metonymy . . . . . . . . . . . . . . . . . . . . . . . . . . 35

4.10 Summing Up This Unit . . . . . . . . . . . . . . . . . . . . . . . . . . . . . . . . . . . . . . . . . . . . . . . . . . . . . . . . . 36

References . . . . . . . . . . . . . . . . . . . . . . . . . . . . . . . . . . . . . . . . . . . . . . . . . . . . . . . . . . . . . . . . . . . . . . . . . 36

Resumo . . . . . . . . . . . . . . . . . . . . . . . . . . . . . . . . . . . . . . . . . . . . . . . . . . . . 39

Referências Básicas e Complementares . . . . . . . . . . . . . . . . . . . . . 43

Atividades de Aprendizagem - AA . . . . . . . . . . . . . . . . . . . . . . . . . . 45
Letras Inglês - English Semantics

Presentation
Dear Student:
It is a pleasure to invite you to study English Semantics with us. We are sure you will love to
find out senses and meaning in the English Language. Read your Caderno Didático very carefully
and do the activities that come together with it. We are sure you will master English Semantics
very easily!
In this course you are going to learn a lot about how words and sentences work out in Eng-
lish. You will also learn about their references and the way we express them in English.
We wish you all the best in your academic and professional career.

Best,
Ana Larissa and Bárbara Orfanó

9
Letras Inglês - English Semantics

Unit 1
Basic Concept

1.1 Introduction
SEMANTICS is the study of MEANING in LANGUAGE. Knowing the meaning of all the
words that make up a language is not sufficient to interpret an utterance, though. We usually
need access to a series of extra-linguistic information about the participants and the context,
their communication intent, the degree of formality of the interactions as well as other elements,
like previous knowledge about a topic, to convey meaning. For this reason, linguists usually dif-
ferentiate between two complementary approaches to the area of meaning production and in-
terpretation.
The first area is concerned with sentence meaning and is the object of semantics. The sec-
ond deals with utterance meaning and is the object of pragmatics (HUTFORD, B; HEASLEY, B &
SMITH, M, 2007; MEYER, 2007).
In order to clarify this, two questions can be addressed:

1. What does it mean?


This question is a request of information and is independent of the participants in a given
interaction and is in the field of Semantics.

2. What do you mean?


This question is a request of information and is dependent of the participants in a given in-
teraction and is in the field of Pragmatics.

We have outlined the basic concepts of Semantics here. Before we move on to the activities
you shall take a look at the reminders, they summarize the main topics covered in this unit. Make
sure that you understand them. Do go back to the unit, if you have doubts before doing the ac-
tivities.

1.2 Sentences, Utterances and Tip

Propositions
Now, analyse this sen-
tence:
“The door is open”
What kind of meanings
can you interpret from
An UTTERANCE is any stretch of talk, by one person, before and after which, there is silence it?
on the part of that person. Someone is informing
you so that there is a
An utterance is the USE by a particular speaker, on a particular occasion, of a piece of lan-
certain door and that it
guage, such as a sequence of sentences, or a single phrase, or even a single word. is open.
Utterances are physical events. Events are ephemeral. Utterances die in the wind. Linguistics Someone is feeling cold
deals with spoken language and we will have a lot to say about utterances in this book. But we and he or she is kindly
will concentrate even more on another notion, that of sentences. asking you to close the
door.
A SENTENCE is neither a physical event nor a physical object. A string of words put togeth-
Someone is asking you
er by the grammatical rules of a language. A sentence can be thought of as the IDEAL string of to leave the room
words behind various realizations in utterances and inscriptions pragmatics (HUTFORD, B; HEAS-
LEY, B & SMITH, M, 2007; MEYER, 2007).

11
UAB/Unimontes - 5º Período

1.3 Meaning and Dictionary


Glossary A DICTIONARY can be thought of as a list of the meanings of words, of what words mean.
A Semântica é o estudo But…could one make a list of what speakers in general mean?
do sentido na língua. The answer is NO because speakers may mean different things on different occasions, even
when using the same words, as in the sentence: “The door is open”.
Enunciado (utterance): So, it is important to understand two basic definitions: speaker meaning and sentence or
é um segmento de fala
concretamente produzi- word meaning.
do por um falante, con- SPEAKER MEANING is what a speaker means, or wants to convey, when he uses a piece of
tém sentido completo. language.
É, normalmente, objeto SENTENCE MEANING (or WORD MEANING) is what a sentence (or word) means, i.e. what it
de análise semântico counts as the equivalent of in the language concerned.
-pragmática.
This makes us come to two important definitions in our course. The definitions of sentence
Período ou frase (sen- and utterance:
tence): é um conjunto SENTENCES are abstract grammatical elements. Utterances are concrete strings of words.
de palavras que seguem Semantics is part of our grammatical competence and usually focuses on decontextualized
um padrão gramatical
meaning, while pragmatics focuses on contextualized meaning. So, the study of meaning, or
e que contém sentido
completo. É, normal- SEMANTICS, has proven to be one of the more challenging levels of linguistic structure for lin-
mente, objeto de guists to describe. A relatively simple word such as the noun CHAIR, is a term to which the Oxford
análise sintática. English Dictionary (OED) assigns 16 different meanings.
Consider one of these 16 meanings as defined in the OED and two other dictionaries:

• OXFORD ENGLISH DICTIONARY (OED): A seat for one person (always implying more or less
of comfort and ease); now the common name for the movable four-legged seat with a rest
for the back, which constitutes, in many forms of rudeness or elegance, an ordinary article of
household furniture, and is also used in gardens or wherever it is usual to sit.
• MERRIAM-WEBSTER’ COLLEGIATE DICTIONARY (MW) (11TH ED.): a seat typically having
four legs and a back for one person.
• AMERICAN HERITAGE DICTIONARY OF THE ENGLISH LANGUAGE (AHD): A piece of furni-
ture consisting of a seat, legs, back, and often arms, designed to accommodate one person.

The three dictionaries agree on two characteristics of a chair: that it seats one person and
has a back. While the OED and MW specify that a chair has four legs, the AHD states simply that it
has legs. The AHD also notes that a chair “often [has] arms”, suggesting that arms are optional. The
other two dictionaries say nothing about arms. The OED entry is much more detailed than the
other entries, noting that chairs exhibit “comfort” and “ease”, are “moveable”, and are regarded as
“household furniture.”
Although the definitions in the three dictionaries are similar, there are enough differences
to illustrate the complexity inherent in defining even the simplest notions. For this reason, many
different perspectives for treating meaning have developed within the field of linguistics. Dic-
tionary definitions fall within the province of lexical semantics, an area of linguistics concerned
with the study of the meaning of individual words. Because dictionaries are intended as refer-
ence guides, they do not provide theoretical statements about the nature of lexical meaning.
However, LEXICOGRAPHERS, those who create dictionaries, have developed methodologies for
discovering the meanings of words and most effectively presenting these meanings to users of
dictionaries.
For this reason, modern lexicographers have abandoned handwritten citation slips created
by thousands of individuals and have turned instead to collecting examples automatically from
very large corpora. For instance, the publisher Harper-Collins created the Collins Word Web as
the source for citation files used to create a number of dictionaries that they have published, in-
cluding The Collins English Dictionary (2007). The Collins Word Web is currently 2.5 billion words in
length and contains various kinds of spoken and written English. It is constantly being updated
so that new words entering the language can be detected and included in upcoming editions of
dictionaries.
Advances in software development have also aided in the creation of citation slips. A CON-
CORDANCING PROGRAM can be used on any computerized text to very quickly create a KWIK
(keyword in context) concordance. All instances of chair are vertically aligned so that their use
in context can be easily examined. Although only sentence fragments in which chair occurs can

12
Letras Inglês - English Semantics

be seen, often, only a limited context is needed to determine the meaning of a word. If a larger
context is desired, most concordancing programs allow for the entire sentence or surrounding
sentences to be viewed.
While lexicographers will need to examine many uses of a word to determine its meaning(s), Glossary
the 24 instances of chair in Figure 1 begin to reveal it. Three of the examples point to a chair as a
Dicionário: é uma lista
place to sit: de palavras e o significa-
do delas.
…in his gown, sitting in a chair…. O sentido que o
Enter the friar, sitting in a chair…. falante confere à sua
produção (speaker´s
The back of the chair on which Gennaio is sitting….
menaing): relaciona-se
ao que o falante quer
One example actually provides a definition of a chair: dizer sobre algo.
A chair consists of four legs, a seat,…. O sentido do período
ou da frase (sentence
meaning): é o que o
Another contains a few words, “…carved wooden chair….”, specifying what a chair is made of.
período ou frase sig-
nificam no cnonstruto
Other examples indicate that chair is POLYSEMOUS; that is, that it has more than one mean- geral da língua.
ing. A chair is not simply a concrete object used for sitting, but an abstract noun designating Lexicógrafos (lexico-
someone who is the head of something, or who holds some highly esteemed position at a uni- graphers): são os pro-
fissionais responsáveis
versity:
por coletar evidências
de uso linguístico para
…Gordon Stewart, chair of the Department of History,…. a produção de dicio-
…B. Watson left his academic chair at The Johns Hopkins University nários.
Linguística de Corpora
(Corpus Linguistics): é
Of course, more examples beyond would be needed to verify this meaning of chair. But as
um ramo da linguística
lexicographers begin isolating multiple meanings of words, they can search for other examples que tem como objeto
to determine how widespread the meanings are. de estudo o levanta-
Words with similar pronunciations but different meanings are often referred to as HOMO- mento e análise da
NYMS. But deciding whether a given word has one or more meanings is often difficult to deter- língua em uso com base
em bancos de dados
mine. A ‘rocking’ chair differs from other chairs because it does not have four legs but two curved
digitais dispoiníveis.
legs that are shaped in a way that permits the chair to move forwards and backwards. A ‘comput- Concordance Programs
er’ chair also moves but typically has four legs with wheels. A ‘beanbag’ chair has no legs or arms e KWK (Key word in
but a flexible area for sitting. context) são ferramentas
All of these chairs are little more than variations on the traditional notion of ‘chair’. For this utilziadas pela linguísti-
ca de corpus.
reason, no lexicographer is likely to list them in a dictionary in a separate entry.
Polissemia (polysemy):
é o fenômeno semânti-
co pelo qual uma mes-

1.4 English Language Dictionaries


ma palavra apresenta
vários significados, reco-
nhecíveis apenas pela
análise do contexto. Ex:
light (luz ou leve).
There are many different kinds of dictionaries: Homínia (homonyny):
MONOLINGUAL DICTIONARIES: Monolingual dictionaries are intended for native speakers é fenômeno semântico
and, as a consequence, focus on a single language (e.g. English, German, French). Some of the pelo qual duas palavras
apresentam significados
more well-known monolingual English dictionaries include the Oxford English Dictionary, Web- diferentes e pronúncia
ster’s Third New International Dictionary, and the American Heritage Dictionary of the English Lan- semelhantes. Ex: flower
guage. For non-native speakers, there are specialized monolingual dictionaries known as learner (flor)e flour (farinha).
dictionaries. For instance, the Cambridge Advanced Learner’s Dictionary and the Collins COBUILD
Advanced Learner’s English Dictionary are written specifically for non-native speakers of English,
and, thus, contain simpler definitions than would be found in a typical monolingual dictionary
and a greater emphasis on vocabulary, such as idioms or phrasal verbs, that give individuals
learning English as an additional language considerable difficulty.
BILINGUAL DICTIONARIES: Bilingual dictionaries focus on two languages and are designed
for individuals who are native speakers of a particular language learning another language as an
additional one. For English speakers, there are English/Spanish dictionaries, English/Italian dic-
tionaries, and so forth.
UNABRIDGED/ABRIDGED DICTIONARIES: The major dictionary makers will periodically re-
lease large unabridged dictionaries from which they will produce smaller unabridged dictionar-
ies that contain a subset of words in the abridged dictionary as well as newer words that have

13
UAB/Unimontes - 5º Período

Glossary entered the language since the publication of the unabridged version. For instance, Webster’s
Há vários tipos prin- Third New International, an unabridged dictionary produced by the G&C Merriam Co. in Spring-
cipais de dicionários, field MA, was released in 1961. Since its the publication, Merriam-Webster has published 11 col-
entre eles: legiate dictionaries - which contain fewer entries than Webster’s Third. At the same time they have
Monolíngue (mono- been updated with newer words than the unabridged version. Because the Webster name is so
lingual): dicionários closely associated with the 19th century American lexicographer Noah Webster, many dictionar-
normalmente direcioan-
dos a falantes nativos ies have been published under the Webster’s name. However, The G&C Merriam Company is the
ou muito proficinetes. only publisher of a Webster dictionary having any connection to Noah Webster’s 1828 dictionary,
Bilíngue (bilingual): di- American Dictionary of the English Language.
recioandos para falantes THESAURUSES: These are dictionaries specialized in providing synonyms for the main en-
de duas línguas dire- tries that they contain. One of the more famous English Thesauruses is Roget’s Thesaurus, pub-
ferentes, pois contém
traduação ou verão. lished in 1852 and written by Peter Roget. Because the name of this dictionary was never copy-
Há ainda dicionários righted, many thesauruses contain the name Roget, even though they are not derivative of the
especializados: por original thesaurus.
exemplo, dicionários de SPECIALIZED DICTIONARIES: Many dictionaries focus on vocabulary specific to a particular
sinônimos (thesauruses) occupation or area of interest. Physicians and lawyers, for instance, can make use of dictionaries
ou dicionários da área
de direito (law), por that define medical and legal terms, such as Tabler’s Cyclopedic Medical Dictionary or Black’s Law
exemplo. Dictionary. Musicians can consult dictionaries of musical terms, such as the Grove Dictionary of
Music and Musicians. Scrabble players have dictionaries specialized in words commonly used in
scrabble games. Since the range of interests is large, so are the number of dictionaries catering to
these interests.
Even though many different kinds of dictionaries exist, most individuals are probably most
familiar with abridged or unabridged monolingual dictionaries: the primary focus of discussion
in this section. The creation of a monolingual dictionary is essentially a two-stage process: deter-
mining the meaning of words by studying their use in context, and, then, crafting definitions of
the words that will be appropriate for the readership of dictionaries.

THESE CONCEPTS ARE


Tip
1.5 Lexical Semantics
KEY. MAKE SURE YOU
UNDERSTAND THEM According to HUTFORD, HEASLEY & SMITH (2007), LEXICAL SEMANTICS has also been stud-
VERY WELL BEFORE YOU
GO AHEAD. ied within linguistics. For instance, one way to describe the meanings of words in a more general
SYNONYMY sense is to categorize the various relationships existing among them: words with similar or iden-
Synonyms are words tical meanings are considered SYNONYMS, those with opposite meanings ANTONYMS. Words
which have a similar with different but overlapping meanings, such as beagle or poodle, can be said to constitute a
meaning. SEMANTIC FIELD. Within semantic fields, certain words will be PROTOTYPES: words more close-
Small, tiny, little are
synonyms. ly associated with the field than other ones. For instance, speakers of American English will re-
ANTONYMY gard a poodle or German Sheppard as more typical type of dog than a Norwegian elk hound. An-
Antonyms are words other more controversial way of characterizing the meaning of words has been done in the area
that have opposite of COMPONENTIAL ANALYSIS. This involves defining words by breaking them down into their
meanings. component parts and assigning them semantic features. On one level, the words puppy and in-
Little and big are an-
tonyms. fant share the feature ‘newly born’. These words differ in that infant has the feature ‘human’, while
HYPONOMY puppy does not. However, this area of semantics has proven to be problematic, primarily because
Words like creature may it is difficult to determine exactly what semantic features are needed.
be seen as a super- While lexical semantics is concerned with the meaning of individual words, larger structures,
ordinate term, with a such as sentences, also have meaning. Functional elements such as subjects and objects have
more general meaning,
whilst cows or mammals SEMANTIC ROLES. In the sentence, The child made a sand castle, the subject of the sentence, the
have specific meanings child, is the AGENT: the person responsible for carrying out the action in the sentence. The direct
which come under, or object, a sand castle, is the PATIENT, the person or thing directly affected by the agent’s actions.
are subordinate to, the Recent work in frame semantics has expanded upon work on semantic roles to describe these
word creature. These roles in terms of the cognitive frames in which they occur. For example, the sentence The woman
subordinate terms are
called HYPONYMS. So, bought a clock would be part of the commercial transaction frame, a frame that includes many
we can see cow as a elements, in the given example a buyer (the woman) and something purchased (a clock).
hyponym of mammal Still, words also have a ‘pointing’ function. This function is known as deixis, a word bor-
which, in turn, is a hypo- rowed from Greek that means ‘to point’ or ‘to show’. In the given example, not only does the word
nym of creature. woman means (‘an adult female’) but it points, or refers, to a particular woman in the external
world. The ability of nouns and pronouns to refer is one type of deixis: REFERENTIAL DEIXIS.

14
Letras Inglês - English Semantics

Other types include TEMPORAL and SPATIAL deixis. For instance, the sentence I walked a mile Glossary
yesterday contains two temporal markers that anchor this sentence in the past: the past tense Sinônimos (synonyms):
marker on the verb walked and the adverb yesterday. Other time frames are indicated by the são palavras que
present tense marker in English as well as the two aspect markers (perfective and progressive). apresentam sentido
Spatial deixis is indicated by prepositions such as in and on or demonstratives such as this or that, equivalente. Ex: salaray
(salário) e wage (salário).
which situate what is being discussed either close to the speaker/writer (“This wine is giving me
Antônimos: são pala-
a headache”) or away from him/her (“That person always bothers me”). vras que apresentam
Finally, language can be used to express the speaker or reader’s perspective on the truth sentido oposto. Ex: high
of what is being said, an area of semantics known as modality. Degrees of certainty can be ex- (alto) e low (baixo).
pressed through modal verbs such as can or may and adverbs such as perhaps, definitely, or Homófonos (homo-
phoes): são palavras
maybe. The sentence I will help you expresses a high degree of certainty, while the sentences Per- que possuem a mesma
haps I will help you or I might help you indicate a much lower degree of certainty. pronúncia e grafia e
The various ways that meaning has been studied shows exactly what is meant by the notion sentidos diferentes.
of ‘meaning’: what philosophers of language often describe as “what it means to mean” (HUT- send (enviar) e sand
(areia).
FORD, HEASLEY & SMITH, 2007; MEYER, 2007).
Homógrafos (homo-
graphs): são palavras
que possuem grafia

1.6 How Semantics Operates: the


idêntica, pronúncia
idêntica ou não e senti-
dos diferentes. Ex: bear
(urso) e bear (suportar,

Two Levels aguentar).


Isotopia (isotopy): con-
junto de palavras que
contém a mesma raiz
semântica. Ex: digital,
Semantic investigation operates at two levels: word and sentence level. The first explores dígito e digitalização.
the relationships words have with each other within a language system, their sense. That can be Hiponímia
defined in terms of SYNONYMY, ANTONYMY, POLYSEMY, HOMONYMY and HYPONYMY. (hyponymy): uma
As we remember from Saussure’s theory, since the relationship between words and their ref- relação hierárquica
erents is merely symbolic – they are signs – each word derives a meaning not from the real world entre termos, na qual
o sentiudo de um está
but from its existence within a semantic field of related signs. contido no sentido do
At the WORD LEVEL, Componential analysis breaks down the meaning of a word into com- outro. Ex: apple (maçã)
ponents. For example, the components of the word man would be: + human + adult + male. e fruit (fruta).
Using these components, semanticists build grids which define the words of a particular field Protótipo (Prototype):
são palavras comumen-
according to the presence or absence of a particular component. Of course, grammatical words te associadas a um cam-
such as and, but, for do not lend themselves to this analysis. But, above all, the elements men- po semântico específi-
tioned could be endlessly broken down into smaller ones. So this method can be useful as a co. Ex: o termo cachorro
means of classification but not as a theory of meaning. (dog) está mais próximo
At the SENTENCE LEVEL, semanticists are mainly concerned with the truth value of linguis- do campo semântico de
animais domésticos do
tic expressions. que o termo wolf (lobo),
They often distinguish between analytic and synthetic truth. A synthetically true statement is embora, do ponto de
true because it is an accurate representation of reality. An analytically true statement is true be- vista biológico, eles se-
cause it follows from the meaning relations within the sentence. jam muito semelhantes.
Papéis semânticos (se-
LOGICAL SEMANTICS or TRUTH CONDITIONAL SEMANTICS draws mainly on propositional mantic roles) - agente
logic and is interested above all in the logical connectives of English. (agent): pessoa ou coisa
This kind of analysis implies a correspondence between language and reality, but some responsável pela ação
semanticists do not believe in this correspondence and argue that language creates reality. enunciativa; paciente
COGNITIVE SEMANTICS understands language as part of our general cognitive ability and (patient): pessoa ou
coisa afetada pela ação
pays special attention to metaphor pragmatics (HUTFORD, B; HEASLEY, B & SMITH, M, 2007; MEY- enunciativa. Ex: I closed
ER, C. Introducing English Linguistics . London: Longman. 2007). the door (Eu fechei a
porta). Eu (I) agente;
porta (door) paciente.
Dêixis Referencial (re-

1.7 Sense Relations ferential deixis): caráter


demonstrativo da lin-
guagem, que recupera
elementos anteriormen-
te ou posteriormente
ANTONYMY is a sense relation between words which are opposite in meaning. citados no discurso. Por
There are various forms of antonymy. exemplo: Paulo comeu
In GRADABLE ANTONYMS there can be degrees of opposition (wide/narrow, old/young/, o bolo. Ele estava com
tall/short). In this case the definition changes according to the REFERENT and there is usually a fome. Ele refere-se a
MARKED (young) and and UN-MARKED term (old ex. She is 16 years old). Paulo.

15
UAB/Unimontes - 5º Período

In COMPLEMENTARY ANTONYMS the opposition between the terms is absolute (alive/


dead). RELATIONAL ANTONYMS are not either/or but there is a logical relationship between
them (above/below, husband/wife)
HOMONYMY is a relation between words which have the same form, but unrelated senses.
HOMONYMS can have the same phonological or graphical form, or both. If they have the
same phonological form, they are called HOMOPHONES (sight/site).
If they have the same graphical form, they are called HOMOGRAPHS (lead: metal and lead:
conduct). Some of them are both homophonic and homographic (mail).
POLYSEMY is a sense relation in which a word, or lexeme, has acquired more than one mean-
ing often because of its metaphorical use or because it can refer to abstract or concrete referents.
Glossary Sometimes homonymy is difficult to distinguish from POLYSEMY, but in fact homonyms are
Ambiguidade (ambi-
separate lexical items which happen to have the same form, while in the case of POLYSEMY the
guity): uma palavra ou same lexical item has taken up more than one sense. One possibility is to take etymology as a cri-
expressão pode con- terion to distinguish them, but it does not always work (sole), so maybe the best approach is to
ter sentidos variados, look for a common core of meaning (common semes).
dependendo do seu An ISOTOPY is formed by repeating one seme. For example, in “There was a fine ship,
contexto de uso. Ex:
We serve turists (Nós
carved from solid gold / With azure reaching masts, on seas unknown”, the words “ship”, “masts”
servimos turistas). Pode and “seas” all contain the seme /navigation/ (as well as others) and thus create the isotopy /navi-
ser entendido como gation/.
turistas são servidos HYPONYMY is a hierarchical relation between two terms, in which the sense of one is in-
pelos garçons de um cluded in the other [rose (hyponym) /flower (hypernym)].
restaurante. Ou, num
contexto de humor,
CO-HYPONYMS are hyponyms of the same hypernym (rose, lily, daisy) and are incompatible
os turistas podem ser (a rose cannot be a lily).
servidos como um prato There can be various levels of hyponymy (Living things - (Animal)/ Vegetable - Flower - Rose/
do cardápio. Lily/Daisy/ Poppy etc).
SYNONYMY is a relation between words which have a similar meaning (mad/insane, main/
chief/principal). English is particularly rich in synonyms because of the influx on it of various lan-
guages such as Latin, French and Anglo-Saxon.
In fact, words are never totally interchangeable, so synonyms frequently differ stylistically,
they belong to different language registers (mother/mom) or can be combined only with certain
other words, that is they have a collocational range (powerful, mighty, strong).

MULTI-MEANING WORDS
In all languages, words may have multiple meanings. It is very important to consider the
context in each a words is.
Examples:

Argument:
1. Making an argument – expressing a point of view and supporting it with facts and evi-
dence: He presented relevant arguments in favor of the new legislation.
2. Harsh discussion: I had an argument with my boyfriend last night.

AMBIGUITY
Tip Sentences can have different meanings, depending on how we interpret them. We usual-
Competent speakers ly rely on context and background knowledge to get rid of ambiguity but, sometimes, meaning
may consider they must be more clearly conveyed.
know the meaning of Examples:
words or sentences of In the sentence:
a language, however
the student (or the
“Include Your Children When Baking Cookies”, the meaning is “are children going to take
professor) of Semantics part in the activity“ OR “are they going to be baked together with the cookies”? As we can see,
may well be good at meaning always depends on context.
describing meanings, LEXICAL AMBIGUITY depends on HOMONYMY (senses not related) and POLYSEMY (senses
or theorizing about related). Some sentences which contain ambiguous words are ambiguous while others are not,
meaning in general. This
kind of reflection is part
and some sentences which contain no ambiguous words are AMBIGUOUS while others are not.
of language education We have outlined the basic concepts concerning sense relations. Before we move on to the
in general. activities you shall take a look at the reminders, they summarize the main topics covered in this
unit. Make sure that you understand them. Do go back to the unit, if you have doubts before do-
ing the activities.

16
Letras Inglês - English Semantics

1.8 Semantic Field Glossary


Metáfora (metaphor):
processo cognitive pelo
qual um termo, perten-
A SEMANTIC FIELD is an area of meaning containing words with related senses. According cente a um determina-
do campo semântico é
to this theory, meanings of words cluster together to form fields of meaning, which, in turn, clus- transferido para outro.
ter into larger ones (Ex.: veal/chicken/pork - meat - food). O campo original é
Each meaning is defined by the space a word occupies in the field. normalmente referido
The origin of the FIELD THEORY OF SEMANTICS is the lexical field theory introduced by Jost como fonte (source) e o
novo como alvo (target).
Trier in the 1930s. For John Lyons (1970) words related in any sense belonged to the same SE- Ex: o termo cabeça
MANTIC FIELD, and the SEMANTIC FIELD was simply a lexical category, which he described as a (head) pode ser usado
LEXICAL FIELD. So, we can say that SEMANTIC FIELDS translate into LEXICAL FIELDS. no sentido de parte de
Field theory is very useful in the contrastive analysis of different languages (wood/glass/ um corpo vivo, como
também no sentido de
types of kinship). Some words can belong to different fields (polysemy). principal. Ex: He is the
Example: head of the departmente
Cat, feline, moggy, puss, kitten, tom, queen and miaow occupy the same semantic field. (Ele é o chefe do depar-
tamento).
Metáfora convencio-
nal (conventional me-

1.9 Metaphor
taphor): é um tipo de
metáfora usada rotinei-
ramente para exempli-
ficar aspectos culturais
do modo como falantes
METAPHOR is a process in which one semantic field of reference is transferred to another. conceitualizam um
The new field is generally referred to as TARGET or tenor, the old one as SOURCE or determinado conceito.
Ex: na maior parte das
VEHICLE. culturas o verbo chegar
Example: (arrive), originalmente
“Time is RUNNING fast”. contém um sentido de
aproximação física. Mas
ele é utilizado, também,
A CONVENTIONAL METAPHOR is a metaphor that is commonly used in everyday language no sentido temporal
in a culture to give structure to some portion of that culture’s conceptual system. For example: (The weekend is arriving:
o final de semana está
The understanding of time as a resource chegando).
Metáfora estrutural
Example: (Structural metaphor):
Time is RUNNING out. é um tipo de metáfora
que ocorre quando um
• The understanding of life as a journey conceito é expresso em
termos de uma estrutu-
Example: ra lingüística semelhan-
It’s time to GET ON with your life. te. Ex: a warm person
(uma pessoa calorosa).
Warm (quente, caloroso)
COGNITIVE SEMANTICISTS, instead, do not make this distinction and consider metaphor as usado para expressar
a natural feature of language and a consequence of the way we think about the world. (LAKOFF uma característica de
& JOHNSON, 1980) distinguish 3 types of metaphor: personalidade.
STRUCTURAL METAPHOR: we map one type of experience onto another. A structural Metáfora orienta-
cional (orientational
metaphor is a conventional metaphor in which one concept is understood and expressed in metaphor): é um tipo
terms of another structured, sharply defined concept: A cold person. de metáfora em que os
An ORIENTATIONAL METAPHOR is a metaphor in which concepts are spatially related to conceitos espaciais es-
each other (I feel down), as in the following ways: tão relacionados entre
si. Prices are higher (os
• Up or down; preços estão mais altos).
• Front or back; Higher (mais alto): usado
• On or off; para indicar elevação de
• Deep or shallow; preços.
Metáfora ontológica
• Central or peripheral. (ontological meta-
phor): é um tipo de me-
An ONTOLOGICAL METAPHOR is a metaphor in which an abstraction, such as an activity, táfora em eu m conceito
emotion, or idea, is represented as something concrete, such as an object, substance, container, abstrato é representado
por algo concreto. Ex:
or person: How did Jerry get out of washing the windows? He broke my heart (ele
We have just outlined the definitions for metaphor. Before we move on to the activities, you quebrou meu coração).
shall take a look at the reminders, they summarize the main topics covered in this unit. Make sure Broke my heart signifi-
cando um sentimento
that you understand them. Do go back to the unit, if you have doubts before doing the activities. de tristeza.

17
UAB/Unimontes - 5º Período

Tip
Chair and table are
heteronyms (in the
1.10 Language Variation and the
lexical field of furniture
terms).
Get on and get off are
Issue of Correctness
directional opposites
with respect to time. English, like most languages, has a number of different DIALECTS. Just as the pronuncia-
Top-down and bot-
tom-up are directional
tion of English VARIES from one dialect to another, so there are also DIFFERENCES in the basic
opposites with respect semantic facts from one dialect of English to another. Note that we are using ‘dialect’ in normal
to the direction of a way in Linguistics, i.e. to indicate any variety of a language, regardless of whether it has prestige
process. or not. In this sense, every speaker, from the London stockbroker to the Californian surfer speaks
Before and after are some DIALECT. It is not the business of semantics to lay down standards of semantic correct-
directional opposites
with respect to time.
ness, to prescribe what meanings words shall have, or what they may be used for. SEMANTICS,
Now, it is time to do the LIKE THE REST OF LINGUISTICS, DESCRIBES. THERE MUST BE NO JUDGEMENTAL VIEW IN
activities for Unit I to THIS DESCRIPTION. (HUTFORD, HEASLEY, & SMITH, 2007; MEYER, 2007 ).
check your comprehen-
sion and practice what
you have learned. Go
ahead!
References
CHERCHIA, Genaro & McCONNEL, Sally. Meaning and grammar: an introduction to semantics.
Massachusetts: The MIT Press, 1995.

HURFORD, James R. & HEASTLEY, Brendan. Semantics: a coursebook. Cambrige: 58 CUP, 1998.

HURFORD, HEASLEY and SMITH. Semantics: a course book. Cambridge: Cambridge University
Press, 2007.

LEECH, Geoffrey. Semantics: the study of meaning. 2. ed. London: Penguim books, 1981.

MEYER, C. Introducing English Linguistics . London: Longman. 2007.

18
Letras Inglês - English Semantics

Unit 2
Reference and Sense

2.1 Introduction
The notions of SENSE and REFERENCE are central to the study of meaning. Every further
unit in this book will make use of one or another of these notions. The idea of REFERENCE is
relatively solid and easy to understand. The idea of SENSE is more elusive: it’s a bit like electricity,
which we all know how to use (and even talk about) in various ways, without ever being sure
what exactly it is.
The REFERENT of an expression is often a thing or a person in the world; whereas the
SENSE of an expression is not a thing at all. In fact, it is difficult to say what sort of entity the
sense of an expression is. Intuitively, it is sometimes useful to think of sense as that part of the
meaning of an expression that is left over when reference is factored out. It is much easier to say
whether or not two expressions have the SENSE. It is like being able to say that two people are in
the same place without being able to say where they are.
The SENSE of an expression is an abstraction, but it is helpful to note that it is an abstraction
that can be entertained in the mind of a language user. When a person understands fully what is
said to him, it is reasonable to say that he grasps the sense of the expressions he hears in a prag-
matic way (HUTFORD, B; HEASLEY, B & SMITH, M, 2007; MEYER, C, 2007; KRAHMER, 1998).

Examples:
When Helen mentioned ‘the fruit cake’, she meant that rock-hard object in the middle of the
table (reference). Glossary
When Albert talks about ‘his former friend’ he means me (reference)
Sense (sentido): é uma
• Daddy, what does unique mean? (sense) abstração, um sentido
• Purchase has the same meaning as buy (sense) que atribuímos a uma
expressão como rep-
Look at the following cartoon. It was published in the USA in 2009. Who (or what) do resentativa do mundo
you think the father is REFERRING TO when he says: “This time”! What does the expression real.
Reference (referência):
“this time” MEAN? refere-se à propriedade
dêitica da linguagem,
que aponta para
◄ Figure 1: Sense and um pessoal objeto,
reference sentimento ou local no
Source: Available mundo.
<http://kingofgng.com/
eng/2008/11/24/barack
-obama-comic-strips-
collection/>. Accessed in
sep. 2010.

The father is possibly REFERRING TO Barack Obama, the first black man to become the
President of the United States. He probably means that, after years of discourse about equality

19
UAB/Unimontes - 5º Período

in the USA, black people can finally be really considered equal to white people in terms of the
opportunities they can have. To understand the charge, you need to grasp both REFERENCE and
Tip
SENSE.
When you look up the
MEANING of a word in
a dictionary, what you
find there, are not ref-
erents, but expressions
with the SAME SENSES.
2.2 Cataphoric and Anaphoric
Glossary
References
Anaphoric reference
(referência anáfora): CATAPHORIC REFERENCE
é uma recuperação de A CATAPHORIC REFERENCE unit refers to another unit that is introduced later on in the
um elemento anterior- text/speech. To understand the unit referred to by a CATAPHORIC REFERENCE, you would need
mente mencionado to look ahead in the text/speech. CATAPHORIC means reference forwards in the text. Sometimes
no discurso. Ex: Allan a pronoun such as he, she, it finds its reference in the following context of the text.
acabou de chegar. Ele
está esperando lá fora. Example:
Ele refere-se a Allan, When he arrived, Selton was surprised to see the door open (He refers to Selton. He came
anteriormente mencio- first in the text).
nado no discurso. When I first met him, John Smith was wearing a very ugly T-shirt.
Cataphoric reference
(referência catafórica):
significa que um termo

2.3 Anaphoric Reference


usado no discurso é
referido ou pode ser
identificado poste-
riormente. Ex: Ele está
esperando por você lá
fora. Você sabe que o An ANAPHORIC REFERENCE unit, on the other hand, refers to another unit that was intro-
Allan odeia esperar. Ele duced earlier on in the text/speech. To understand the unit referred to by an ANAPHORIC REF-
refere-se a Allan, poste- ERENCE, you would need to look back in the text/speech. ANAPHORIC REFERENCE means
riormente mencioando reference backwards in the text. A personal pronoun, for example, often has an anaphoric refer-
no discurso.
ence. (HUTFORD, B; HEASLEY, B & SMITH, M, 2007; MEYER, C., 2007; KRAHMER, 1998).
Exophoric reference
(referência exofórica): Example:
o termo mencionado Selton is so stressed out about the assignment he is talking about it right now. (He refers to
refere-se a um elemento Selton; Selton came first in the text)
fora do discurso. Pode This is the girl who I told you about. (Who refers to girl; Girl came first)
ser recuperado por
meio de vários elemen-
tos a ele refelacionados,

2.4 Exophoric Reference


de acodo com a cultura
ou conhecimento de
mundo.

EXOPHORIC WORDS refer to something outside the text. In linguistics, exophora is refer-
ence to something extralinguistic, i.e., NOT IN THE SAME TEXT, and contrasts with endophora.
Exophora can be DEICTIC, in which special words or grammatical markings are used to make
reference to something in the context of the utterance or speaker.
For example, pronouns are often EXOPHORIC, with words such as this, that, here, there, as in
that chair over there is John’s, said while indicating the direction of the chair referred to.
Given Did the gardener water those plants?, it is quite possible that those refers back to the
preceding text, to some earlier mention of those particular plants in the discussion. But it is also
possible that it refers to the environment in which the dialogue is taking place; to the context of
the situation, where the plants in question are present and can be pointed to, if necessary. The in-
terpretation would be those plants there, in front of us. This kind of reference is called EXOPHORA,
since it takes us outside the text altogether. EXOPHORIC REFERENCE is not COHESIVE, since it
does not bind the two elements together into a text.
The study of SENSE demands, as you may have noticed, a degree of idealization of the facts
about meaning. In other words, sometimes we claim to be more certain than we perhaps should
be about questions like ‘Does this expression have the same sense as that one?’ (HUTFORD, B;
HEASLEY, B & SMITH, M, 2007; MEYER, 2007; KRAHMER, 1998).

20
Letras Inglês - English Semantics

2.5 Reference and Sense in Tip


Before you go ahead,
make sure you under-

Descriptive Meaning stand these key con-


cepts very well:
ANAPHORIC REFER-
ENCE: It is an instance
Words are connected with certain concepts and meanings. When a word is successfully of an expression
communicated, it is recognized by the addressee(s) and it triggers, in the mind of the addressees, referring to another.
The referent comes first:
the concept it is connected with. For example, if I hear someone telling me: Hello, Rafael and Davi Allan has just come. He
have just arrived and want to talk to you, I can realize that the register of language is informal be- is waiting outside.
cause of the use of the word hello. I can also understand the message conveyed (some people CATAPHORIC REFER-
have arrived and want to talk to me) and I can also picture in my mind the image of Rafael and ENCE: It means that a
Davi, once I know whom they are. This is how the process of SENSE and REFERENCE operates in word in the text refers
to another and that you
our minds. have to look forward to
find the referent: He is
waiting for you outside.

References
You know that Allan
hates waiting.
EXOPHORIC REF-
ERENCE: It refers to
information from
CHERCHIA, Genaro & McCONNEL, Sally. Meaning and grammar: an introduction to semantics. outside the text. In the
Massachusetts: The MIT Press, 1995. following traditional
song, the word you may
HURFORD, James R. & HEASTLEY, Brendan. Semantics: a coursebook. Cambrige: 58 CUP, 1998. refer to many different
people in the actual and
HURFORD, HEASLEY and SMITH. Semantics: a course book. Cambridge: Cambridge University fictional situation:
Well in my heart you are
Press, 2007. my darling
And my gate you will
LEECH, Geoffrey. Semantics: the study of meaning. 2. ed. London: Penguim books, 1981. come in (...)
MEYER, C. Introducing English Linguistics . London: Longman, 2007

21
Letras Inglês - English Semantics

Unit 3
Word Meaning in Dictionaries

3.1 Introduction
Dictionary definition - According to Hurford, Heasley and Smith (2007, p. 195), a dictionary
is a central part of the description of any language. It usually gives at least three kinds of informa-
tion:
1- Phonological information explaining, for example, how words are pronounced;
2- Grammatical (syntactical and morphological) containing information about its part of
speech, (adjective or noun), inflections (plural number, present tense);
3- Semantic information containing information about the word meaning. See example
from box 1.

Box 1 - Example taken from Cambridge dictionary online


Definition
problem noun

/’prɒb.ləm/ /’prɑː.bləm/ n [C]

A situation, person or thing that needs attention and needs to be dealt with or solved
financial/health problems.
Our main problem is lack of cash.
I’m having problems with my computer.
No one has solved the problem of what to do with radioactive waste.
The very high rate of inflation poses/presents (= is) a serious problem for the govern-
ment.
When is the government going to tackle (= deal with) the problem of poverty in the in-
ner cities?
[+ -ing verb] Did you have any problems (= difficulties) getting here?
I’d love to come - the only problem is I’ve got friends staying that night.

A question in mathematics which needs an answer.


We were given ten problems to solve.

Source: <www.cambridgedictionaryonline.com> acessed October 2010.

3.2 The Organization of


Dictionaries
A good dictionary should tell users what words mean. In relation to that, it can be said that
dictionaries also describe the senses of predicates (see chapters 1 and 2), for a revision on predi-
cates). However, within a modern linguistics approach, it is necessary to bear in mind that dic-
tionaries are also responsible for representing important aspects of the mental knowledge of its
users and presenting definitions that would be ratified by any typical native speaker of that par-
ticular language (HURFORD, HEASLEY and SMITH, 2007).

23
UAB/Unimontes - 5º Período

Glossary In addition to that, one important characteristic of dictionaries is the interconnectedness of


Dicionário: é parte definitions, as illustrated in picture 5.1. This interconnectedness is unavoidable, due to the fact
central na descrição de that dictionary writers’ main purpose is to define, as completely as possible, the knowledge a na-
qualquer língua. A en- tive speaker has about all the sense relations among predicates (see section 3 on sense relations).
trada de um dicionário This interconnectedness is also evident in the work of Hurford, Heasley and Smith, (2007), who
apresenta, pelo menos, assert that dictionary definitions also rely on a circularity approach in order to completely define
três informações
básicas: (a) fonológica: the meaning of the words.
indica como as palavras See figure 2, as an example of the circularity mentioned:
são pronunciadas; (b)
gramatical: apresenta
a organização das
palavras no discurso Figure 2: Circularity ►
por meio de sua função Mentioned
sintática; (c) semântica:
apresenta informações Source: Available <www.
thesaurus.com>. Acces-
sobre o significado das sed in oct. 2010.
palavras em contexto
de uso.

Another important issue related to dictionaries is the matter of precision. A good dictionary
should reveal a high standard of that. However, such characteristic has proved to be difficult to
achieve.

3.3 Encyclopedia Versus


Dictionary Meaning
Another important fact influencing the organization of a dictionary is the debate between
encyclopedia and dictionary. In order to explain this issue, it may be beneficial to start with brief-
ly mentioning some of the characteristics of encyclopedias and how they are related to diction-
aries.
Tip Most dictionaries bring together the characteristics of encyclopedias, giving information
not relevant to the sense of words. Observing the relationship between encyclopedic informa-
A dictionary describes
the senses of predicates; tion and dictionary definitions, Hurford and his colleagues state that a descriptive semanticist is
An encyclopaedia interested in information about words which can interpret analytic sentences (the walrus is an
contains factual infor- animal) or contradictions (the walrus Is not an animal). He also calls our attention to the fact that
mation of a variety of any other information is not semantic, but encyclopedic, and he also observes that such distinc-
types, but generally no tion can also be related to the notion of ‘narrower sort of dictionary information’ and encyclo-
information specifically
on the meanings of pedic information (as mentioned before) in characterizing the meaning of a linguistic expression.
words. The importance of the difference between the two is still being discussed by semanticists. Hav-
ing mentioned the characteristics of encyclopedic information and dictionary information, the
following section will concentrate on how semantic information is represented in dictionaries.

3.4 Sense Relations: Representing


Semantic Information
The aim of this unit is to present how semantic information is represented in diction-
aries. Considering that a dictionary is a list of predicates and their senses, it is worth look-

24
Letras Inglês - English Semantics

ing at how sense relations are structured in dictionaries. One cannot forget that a semanticist Tip
dictionary-writer is interested, for example, in the sense relation of the words, thus, terms like Dictionary definition:
hyponymy, antonyms, synonyms, etc., prove to contribute in the definition given by dictionaries, Its central role is to de-
when defining some words. scribe the language;
In order to represent the semantic information present in dictionaries, first, it is necessary Dictionary organ-
to introduce the central idea of a meaning postulate. According to Hurford, Heasley and Smith ization: Dictionaries
represent definitions
(2007, p. 204), ‘a meaning postulate is a formula expressing some aspect of the sense of a predi- that should be ratified
cate. It can be read as a proposition necessarily true by virtue of the meaning of the particular by native speakers of
predicate involved.’ the target language;
Encyclopedia and
Example: dictionary meaning:
While dictionaries
HUMAN BEING: are responsible for
One-place describing the meaning
synonym of man1 of predicates, encyclo-
pedias are responsible
MAN 1: for giving factual infor-
mation;
One-place Meaning postulate:
synonym of HUMAN BEING Expresses the aspects
of the sense of the
MAN 2: predicates.
One-place
hyponym of MALE
hyponym of ADULT
hyponym of HUMAN BEING

As mentioned before, the predicates of a language all fit into a complicated network of Tip
interrelationships. A predicate is usually related through this network to other predicates. De-
Symmetry X asym-
spite all the connections between predicates, semanticists want the presentation of information metry: Symmetry is
to be economical including the minimum number of meaning postulates. However, it is import- when the predicates
ant to have a general view of the most important properties of a predicate. have a similar mean-
ing, while asymmetry
is when the predicate
have different mean-

3.5 Properties of Predicates ings;


Reflexivity X irreflex-
ivity: Reflexivity is
when the meaning of
a predicate refers back
• Symmetry and Asymmetry to the meaning of its
The dictionary can give the information that a predicate is symmetric, in the form of a mean- referring item. Irreflexiv-
ing postulate. When predicates have a similar meaning, we say that they are symmetric. Sym- ity is when the meaning
metry is the opposite of asymmetry, thus when the meaning of the predicates are different, we of the predicate is not
say that they are asymmetric. See the following example of a symmetry predicate. related to the meaning
of the referring item;
Example: John is married with Anne is symmetric of Anne is married with John. Now see an Transitivity X
example of an asymmetric predicate. intransitivity: Transitiv-
Example: Lucy is more intelligent than Martha is asymmetric of Martha is more intelligent than ity is represented when
Lucy. compound sentences
are compatible with
each other. When this
• Reflexivity does not happen, we
Reflexivity occurs when the meaning of the predicate refers back to the meaning of the re- call it intransitivity.
ferring item. See the following sentence as an example of reflexivity.
Example: Martha is quite as taller as herself
Following the idea of symmetry and asymmetry, we also have the idea of irreflexive predi-
cate. We have an irreflexive predicate when the referents correspond to a contradiction.
Example: Paul is different from himself.

• Transitivity
Transitivity occurs when compound sentences are compatible with each other.
Take as an example the following sentence:
Monica is in her office and her office is in this building means that Monica is in this building. So
the predicate in is transitive.

25
UAB/Unimontes - 5º Período

Now pay attention to the next example:


Glossary Lucy is the mother of Linda and Linda is the mother of Martha is incompatible with Lucy is the
Propriedades do mother of Martha, so mother of is intransitive. Another important thing to bear in mind is that
predicado - simetria X just as asymmetry and irreflexivity correspond to symmetry and reflexivity, so intransitivity cor-
assimetria responds to transitivity.
Simetria: ocorre
quando os predicados
têm um significado
semelhante. Ex: João
é casado com Maria.
Maria é casa com João.
3.6 Introduction to Derivation
Assimetria: ocorre
quando os predicados
têm significados difer- When we form new words according to a regular pattern on the basis of pre-existing words,
entes. Ex: Lucia é mais we are using the rules of derivation. New words are formed by combining existing words with
inteligente que Márcia meaningful units smaller than words or with other existing words.
(não se pode dizer que
Márcia é mais inteligen- Morpheme
te que Lúcia).
Propriedades do The meaningful units smaller than words, mentioned before, are called by linguists mor-
predicado – reflexivi- phemes. According to Hurford, Heasley and Smith (2007, p. 226), a morpheme is a minimal unit
dade X irreflexividade of word building that combines a minimal unit of meaning with a minimal linguistic form that
Reflexividade: o sig- carries this meaning. Morphemes are considered to be the ‘building blocks’ of a language and
nificado do predicado they are very important, for example, for the concept of prefixes and suffixes.
refere-se integralmente
ao elemento a ele
relacionado. Ex: Marta Prefix
parece mais alta do que
ela mesma.
Prefix Function Example
Irreflexividade: o sig- mis- indicate the action is misunderstand
nificado do predicado
incorrect misunderstanding
não está relacionado ao
elemento a ele relacion- over- indicates exaggeration overwork
ado. Há, nesse caso, um
oversleep
fenômeno de contra-
dição. Ex: Paulo está out- it means that the action is done in a better to grow-----outgrow
diferente dele mesmo.
or deeper way to run---- outrun
Propriedades do
predicado – transitivi- un- negative/opposite from the original believable---unbelievable
dade X intransitivi- necessary--- unnecessary
dade
Transitividade: ocorre il- they form the opposite from the original logical---illogical
quando sentenças com-
im- word perfect---imperfect
postas são compatíveis
umas com as outras. in-
Ex: Mônica está no ir-
escritório dela, que fica
neste prédio. Nesse dis- it forms the opposite from the original to obey---disobey
caso, ambos, Mônica word to like---dislike
e o escritório estão no
mesmo local. half- indicates almost asleep---half asleep
Intransitividade: semi- conscious---semi-conscious
ocorre quando sen-
tenças compostas são
self- it indicates that the action is done to/for control---self-control
incompatíveis entre the own person confident---self-confident
si. Ex: Marta é mãe da
Sofia. Nesse caso, não Another unit smaller than an actual word is the prefix re- in the word remake, the word re is
poderíamos dizer que
also a morpheme, because it combines a minimal meaning (something like ‘repeat
Sofia seja mãe de Marta.
the activity described by the verb it is attached to’) with a minimal linguistic form.

Suffix
The word teacher, for example, is formed by attaching the suffix –er before the root word
teach and the derived word remake is formed by attaching the prefix re- after the root word
make. The root words in both examples correspond to the meanings of the derived words. Thus,
in summary, prefixes are always attached before the root morpheme and suffixes after the root
morpheme. Due to their importance and relevance to the semantics of the English Language,
we shall include here a more detailed grid with the most common prefixes and suffixes, for your
reference.

26
Letras Inglês - English Semantics

Prefixes

Suffix (1): Transforming verbs into nouns


Glossary
Suffix verb noun
Derivação (deriva-
ion- to invent invention tion): é o fenômeno
to limit limitation pelo qual ovas palavras
to compose composition podem ser foram, a
partir de um padrão
ment- to develop development regular, formado por pa-
to improve improvement lavras pré-existentes.
Morfemas (mor-
ing ( when the centre to teach teaching phemes): são a menor
of a noun phrase) to lean learning unidade de análise
Example: Her teach- morfológica.
ing is inspiring. Prefixo (prefix): são
formas presas adicio-
ance/ence to appear appearance nadas ao início das
to depend dependence palavras e que colabor-
to rely reliance am para a formação do
sentido. Ex: This is un-
grammatical (un- indica
negação, formando a
Suffix (2): Transforming verbs into nouns palavra nao-gramatical-
ungrammatical).
er- or- Suffix (sufixo): são for-
to teach-teacher to direct-director mas presas, adicionadas
to play-player to govern-governor ao final das palavras, e
to read-reader que colaboram para a
to hang-hanger formação do sentido.
to blend-blender Ex: He is careless (less
indica sem, faltando,
formando a palavra
descuidado- careless)
Suffix (3): Adjective into adverb
ly-
quick quickly
increase increasingly
beautiful beautifully

Tip
3.7 Compound Nouns Derivation: When we
form new words using
pre-existing words;
Morpheme: Meaning-
Derived words are also represented by compound nouns. Compound nouns are formed
ful units smaller than
when we join two pre-existing words. Examples of derived words as the following: checkout, words;
doorknob, spaceship and babysit are compounds, as they consist of two pre-existing root words. Prefix: A unit smaller
than an actual word
placed at the beginning
of the root word;

3.8 Types of Derivation Suffix: A unit smaller


than an actual word
after the root word;
Compound nouns: The
• Inchoative combination of two
pre-existing words;
An inchoative type of derivation denotes the beginning, or coming into existence, of some Types of derivation
state. Example: Light Dark (adjective) denotes a state. Enlighten (intransitive verb), as in: The room Inchoative: The begin-
has enlightened, is the corresponding inchoative form, because it denotes the beginning of a ning of a state;
state of lightness. Causative: An action
that causes something
to happen;
• Causative Resultative: A state
A causative form corresponds to an action which causes something to happen. Example: resulting from some
close (transitive verb) is the causative form corresponding to close (intransitive verb). If one closes action.

27
UAB/Unimontes - 5º Período

a door, for example, one causes it to close (in the intransitive sense of close). It is important to no-
tice that in English zero-derivation is the most common device for producing causative forms, al-
though causatives are also frequently formed by adding the suffix -en to the non-causative root.

• Resultative
A resultative form denotes a state resulting from some action. Example: Broken (used as
an adjective) is the resultative form corresponding to break (transitive verb). The state of being
broken results from the action of breaking.
Example: That broken door was broken yesterday by the boys.

The figure 3 demonstrates graphically the concepts mentioned.


Figure 3: Types of
derivation
Source: HUTFORD, HEAS-
LEY and SMITH 2007, p.234 ►

Glossary
Tipos de derivação
Inchoative: tipo de
derivação indica o
surgimento de um
estado. Ex: The room
has enlightened with her
presence.
Causative: tipo de
derivação que indica
3.9 Participant Roles
que a uma determinada
ação causou outra. Ex: • Agent
The strong wind closed
the door. The agent of a sentence is the person carrying out the action described.
Resultative: indica o Example: My mother in: My mother closed the window.
resultado de uma ação.
Ex: That window was • Affected
broken by the ball. The affected participant is the thing (usually not a person, although it can be) upon which
Compound nouns: rep-
resentam a combinação the action is carried out. In many cases, the thing is changed by the action, the window in the
entre dois substantivos example given.
pré-existente que forma
um terceiro substantivo. • Instrument
Ex: pet-shop. The instrument is the thing (usually not a person) by means of which the action is carried
out.
Example: They found the place with a map.
Tip Check the following grid for a brief summary of the three roles given:
Agent: The person
carrying out the action;
Affected: The thing They Agent
affected by the action;
Instrument: The thing the place Affected
related to the action
a map Instrument
taking place;
Location: Refers to the
place where the action • Location
takes place; The role of location is played by any expression referring to the place where the action de-
Beneficiary: The person
scribed by a sentence takes place.
who benefits from the
action;
Experiencer: The per- • Beneficiary
son who experiences The beneficiary is the person for whose benefit or to whose detriment the action described
the action; by the sentence is carried out. It is usually assumed that the beneficiary, if mentioned, is distinct
Theme: Thing or person
from both the agent and the affected. The following figure gives a better view of the relationship
perceived by the experi-
encer. among: affect, beneficiary, location and agent. This process is illustrated below:

28
Letras Inglês - English Semantics

Glossary
Participant roles
◄ Figure 4: Participant (papéis dos partici-
roles pantes)
Source: HUTFORD, Agent (agente): pessoa
HEASLEY and SMITH, 2007, ou coisa responsável
p.249 pela ação representa-
da pelo verbo. Ex: My
mother baked the cake.
O agente é minha
mãe (my mother),
responsável pela ação
de assar.
Experience Affected (afetado): é a
The experiencer is a person who is mentally aware of, perceives, or experiences the action or coisa ou pessoa afetada
state described by the sentence, but who is not in control of the situation. (experiencer charac- pela ação. Ex: The boys
teristics can also sometimes be attributed to animals.) broje the window. O ser
Example: The girls listened to the entire story. afetado é janela (win-
dow), afetada pela ação
de quebrar.
Theme Instrumet (instru-
The theme participant is a thing or person whose location is described, or a thing or person mento): é o elemento
that is perceived by an experiencer. mencionado como
Example: The girls listened to the entire story. meio pelo qual a ação é
realizada. Ex: They fond
the place with a map.
The figure 5 shows the relationship between experiencer and theme. (map-mapa refere-se ao
meio pelo qual o local
foi encontrado.
◄ Figure 5: relationship Location (localização):
between experiencer é o elemento que indica
and theme. o local onde uma ação
Source: HUTFORD, foi realizada. Ex: They
HEASLEY and SMITH, 2007, visited London (Eles
p.251 moram em Londres). In
London (em Londres)
indica local.

3.10 Summing up This Unit


Beneficiary
(beneficiário): indica
o ser ou coisa que se
beneficiou ou recebeu
a ação verbal. Ex: I
We have covered the main concepts related to the definition of words. First, we briefly dis- gave him the post cards
cussed the basics concerning the structure of a dictionary. Then, we called attention on how the (o termo him-ele é o
beneficiário da ação de
semantic information is represented in the format of a dictionary. Finally, we focused on deriva-
dar).
tion and the role played by the participants involved in the construction of meaning. Experience (experien-
You can now do the activities (‘Atividades de Aprendizagem’) at the end of this booklet. ciador): indica o ser que
experiência a ação ver-
bal. Ex: The girl listened
to the entire story. Nesse

References caso, a garota (girl)


experienciou a ação de
ouvir.
Theme (tema): é a coisa
HURFORD, HEASLEY and SMITH. Semantics: a course book. Cambridge: Cambridge University ou pessoa experien-
ciada pelo experien-
Press, 2007. ciador na ação verbal.
Ex: The girl listened to
LEECH, G. Principles of pragmatics. London: Longman, 1983 the entire story. Nesse
caso, a estória (story) é
LEVINSON, S. Pragmatics. Cambridge: Cambridge University Press, 1983 o tema.
McCARTHY, M.J. Applied Linguistics. Cambridge: Cambridge University Press, 1998

OLIVEIRA, NÁDIA, A. de. Para ler em inglês: desenvolvimento da estratégia de leitura. Belo Hori-
zonte: O Lutador, 1988.

29
Letras Inglês - English Semantics

Unit 4
Interpersonal and Non-Literal
Meaning

4.1 Introduction
In this unit we are going to expand the focus of our previous discussion on meaning, mov-
ing beyond the concepts of sense, reference, and logic by concentrating on aspects of interper-
sonal meaning, such as speech acts and various kinds of inference, including conversational im-
plicature. This sort of meaning goes beyond literal meaning and entailment relationships based
on truth conditions, and involves aspects of the context of the utterance and intentions of the
speaker.

4.2 Speech Act


Speech acts are the acts that we do with words, such as apologizing, requesting, asking, in-
viting and so on. The theory of speech acts came into being with Austin (1962) who differenti-
ated between sentences that describe a state of affairs and sentences that can be considered as
the performance of an act. He used the term ‘constative’ to refer to the former type of sentences
and ‘performative’ to refer to the latter ones. There are several taxonomies for classifying speech
acts; the following is Searle’s (1976) classification of speech acts (also quoted by Levinson, 1983,
p. 240), who recognizes that this taxonomy is not complete or exhaustive:
1. Representatives - which commit the speaker to the truth of the expressed proposition (as-
serting, concluding, etc.);
2. Directives - which are attempts by the speaker to get the addressee to do something (re-
questing, questioning);
3. Commissives - which commit the speaker to some future course of action (promising,
threatening, offering);
4. Expressives - which express a psychological state (thanking, apologizing, welcoming, con-
gratulating);
5. Declarations - which effect immediate changes in the institutional state of affairs and tend
to rely on elaborate extra-linguistic institutions (excommunicating, declaring war, christen-
ing, firing from employment).

4.3 Perlocutions and Illocutions


Austin (1962) points to the existence of three kinds of action within each utterance:
a. Locution - which is the physical act of producing an utterance;
b. Illocution - which refers to the act which is committed by producing the utterance;
c. Perlocution - which is the production of an effect through locution and illocution.
Taking into account Searle’s (1976) taxonomy together with Austin’s (1962) classification of
speech acts, Bach and Harnish (1979, p. 39-55) develop a taxonomy of speech acts that distin-
guishes types of communicative illocutionary acts by the attitudes the speaker expresses in per-

31
UAB/Unimontes - 5º Período

forming them. This taxonomy presents many types of illocutionary acts in detail which are classi-
fied according to the speaker’s intentions:

a) Constatives: Expression of belief together with the expression of an intention that the
hearer forms (or continues to hold).
Example: I declare you husband and wife.

b) Directives: Express the speaker’s attitude towards some prospective action by the hearer:
Example: I insist that you stay for dinner.

c) Commissives: Are acts of obliging oneself to do something specified in the propositional


context, which may also specify the conditions under which the deed is to be done or does not
to have to be done.
Example: I promise I’ll never talk to him again.

d) Acknowledgments: They express certain feelings towards the hearer:


Example: I would like to congratulate you.

4.4 Felicity Conditions


He pointed out that ‘performatives’, in order to be performed successfully, require a set of
‘felicity conditions’. Thus a ‘felicity condition’ is considered the essential appropriate condition for
a speech act to be recognized as intended.
Searle (1969) recognizes four felicity conditions for speech acts: propositional, preparatory,
sincerity and essential. An example from Levinson (1993, p. 240) illustrates the conditions needed
for the successful illocution of requests (S = Speaker; H = Hearer):
Propositional content: Future Act A of H
Preparatory:
1. S believes H can do A

2. It is not obvious that H would do A without being asked


Sincerity: S wants H to do A

Essential: Counts as an attempt to get H to do A


Searle states that while the sincerity condition tells us what the speaker expresses in the
performance of the act, the preparatory condition tells us (at least partially) what the speaker im-
plies in the performance of the act. Thus, in the performance of any illocutionary act, the speaker
implies that the preparatory conditions of the act are satisfied. For example, when one makes a
statement, there is an implication that one can back it up; when one makes a promise, there is an
implication that the thing promised is in the hearer’s interest (SEARLE, 1969, p. 65). On the other
hand, Searle acknowledges that ‘the meaning of the sentence determines the illocutionary force
of its utterances’ (SEARLE 1969, p. 143). This relationship between meaning and form is also high-
lighted by Sinclair’s (1996) study, suggesting that both the form and force of an utterance are
related.
Bach and Harnish (1979) assert that the speaker expresses the attitude towards the prepos-
itional content as well as the intention that the hearer has or forms from a corresponding prop-
ositional attitude. The speaker having the attitude expressed is the mark of sincerity, but sincerity
is not required for communicative success; nor is the hearer’s belief that the speaker has the atti-
tude expressed. Thus, a communicative illocutionary act can succeed, even if the speaker is insin-
cere, and even if the hearer believes he is insincere.
According to Carter and McCarthy (2006), in everyday written and spoken interactions, there
are some speech acts which are performed with great frequency. Some of the most frequent
speech acts mentioned by Carter and McCarthy (ibid.) are summarized in the following grid:

32
Letras Inglês - English Semantics

1. Commands and instructions: Speech acts in which the speaker is in a position to direct the
Glossary
behavior of the listener, and the listener has little or no freedom to negotiate the action: Speech acts (atos de
a) Declaratives with you can; fala): são atos sociais
produzidos por meio da
b) Interrogatives with can/ could/ will/ would you. They soften the request or command; linguagem verbal. Ex:
c) Declaratives with you must: It is used in commands and instructions in declarative clauses. pedir desculpas, saudar,
reclamar, elogiar.
2. Requests: Speech acts in which the speaker desires a particular course of action from the lis- Atos de fala- cate-
tener, but unlike commands or instructions, the listener has a choice to act in the way indicated. A gorias de análise
request may also be concerned with asking for permission to act in a particular way. Representativas: são
atos de fala em que o
3. Warnings: Speech acts in which the speaker states his/her perception of the negative outcome falante compromete-se
of a particular course of action. The listener may choose to heed or not the warning. com a verdade por ele
expressa. Ex: This bag
4. Advice: Speech acts in which the speaker proposes a desirable course of action for the listener is blue (representa uma
or others, or which may include the speaker him/herself. The speaker may choose to heed or not asserção sobre um
the advice. elemento do grupo).
Diretivos: são atos de
5. Suggestions: Speech acts in which the speaker proposes a desirable course of action or a set of fala em que o falante
compele o outro a fazer
options to be considered for the listeners or others, or which may include the speaker him/herself. algo por ele. Ex: Come to
The listener may choose to heed or not the suggestion. my room tomorrow.
Comissivos: são atos
6. Offers: Speech acts in which the speaker volunteers to do something for the listener (or a third de fala pelos quais o
party) or give something to the listener (or a third party). The listener may accept or reject the falante compromete-se
offer. Offers may be offers to do something or offers of physical things (e.g. food, drink). em realizar algo no
futuro. Ex: I Will come
7. Invitations: Speech acts concerned with offering someone an opportunity to do or share some- tomorrow.
thing (usually pleasurable) with the speaker. The listener may accept or reject the invitation. Expressivos: são atos
de fala pelos quais o
8. Permissions: Speech acts concerned with requesting and granting freedom for someone to act falante expressa um
in a particular way. estado psicológico de
agradecimento, elogio,
9. Prohibitions: Denying freedom of action. boas-vindas. Ex: Con-
gratulations! You did a
good job.
Declarativos (ou per-

4.5 Direct and Indirect Form


formativo): são atos de
fala pelos quais o uso
da linguagem modifica
a realidade. Ex: I declare
you husband and wife.
• Direct form
The direct form of an utterance is the illocution indicated by a literal reading of the gram-
matical form and vocabulary of the sentence uttered.

• Indirect form
The indirect form of an utterance is any further illocution the utterance may have.
Examples:
1. Direct form: Can you pass me the butter?
2. Indirect form: The indirect illocution is a request that the hearer pass the salt.

4.6 Inference
An inference is any conclusion that one is entitled to draw from a sentence or utterance.

4.7 Entailments
According to Yule (1996, p. 25), an entailment is something that logically follows from what
is asserted in an utterance. Check the following example:

33
UAB/Unimontes - 5º Período

Example: Mary’s father bought three cows.


The sentence presented is treated as having the entailment that Mary’s father bought some-
thing. All entailments are inferences, but not all inferences are entailments, as stated by Hurford,
Heasley and Smith (2007). Implicature is another kind of inference, distinct from entailment.

Speech acts: Acts we


Tip
4.8 Conversational Implicature
do with words;
Locution and illocu-
tions: They refer to the Grice (1975, p.46) affirms that the meaning of a speaker utterance often conveys more than
act of producing an words literally mean. He develops the theory of implicature to refer to the process in which the
utterance; listener, through implicature, recognizes the meaning of an utterance. He points out that the
Perlocution: Produc- success of this process, which he called ‘conversational implicature’, depends on the hearer’s
tion of an effect on co-operation. This principle of co-operation involves four maxims:
locution and illocution;
Felicity conditions: • Quantity
The essential condition 1. Make your contribution as informative as required (for the current purposes of the ex-
for a speech act to be change);
recognized; 2. Do not make your contribution more informative than is required.
Direct X indirect
forms: A direct form is
the illocution read lit- • Quality
erally while the indirect 1. Do not say what you believe to be false;
form corresponds to 2. Do not say that, if you do not have adequate evidence.
further interpretations
of that illocution. • Relevance
Inference: Further con-
clusions people might 1. Be relevant.
come to from sentences
or utterances; • Manner
Entailments: Logical in- 1. Avoid obscurity of expression;
terpretations in relation 2. Avoid ambiguity;
to any utterances;
Implicature: The pro- 3. Be brief (avoid unnecessary prolixity);
cess the listener goes 4. Be orderly.
through to understand In conversational interaction, the notion of implicature is triggered when these maxims
an utterance; are not respected. According to Grice (ibid.), the hearer infers meaning, at some level, from the
Cooperative Principle: utterance and from the divergence from it. The problem regarding this theory is pointed out by
Conversational rules
governing conversation: Levinson (1983) and refers to the possibility of having other maxims such as the ‘tact maxim’, ac-
quantity, quality, man- knowledged by Leech (1983), which refers to the perceived cost-benefit scale in directive and
ner and relevance. commissive acts. On the other hand, the maxims are vague, and it is not always easy to deter-
mine in all interactional situations how much information one must convey in order to respect
the maxim of quantity. The maxims also appear to be culturally specific, presenting degrees of
appropriateness which are culturally bound. Aijmer (1996, p. 25), referring to the conventionaliz-
ation of speech, points out the importance of acknowledging culture (see also Blum-Kulka et al.
1989; Nattinger and DeCarrico 1989; Wierzbicka 1991).Take a look at the following example:

Example
Charlene: I hope you brought the bread and the cheese.
Dexter: Ah, I brought the bread.

Charlene has to understand, after hearing Dexter utterance, that Dexter is cooperating and
aware of the quantity maxim. Note that he did not mention the cheese. If he had brought the
cheese, he would have said, because he would be in accordance to the quantity maxim. He must
intend that she understands that what is not mentioned was not brought. Thus, Dexter has con-
veyed more than he said via a conversational implicature. Due to the fact that implicatures are
most of the time realized by inferences, we shall take a look at them.

34
Letras Inglês - English Semantics

4.9 Non-literal Meaning: Idioms,


Metaphor, and Metonymy
Introduction
So far, we have looked at the literal meaning of words which is related to two basic things:
1. The meanings of words and sentences are independent of the context or occasion;
2. That the meaning of a composite expression is essentially compositional5.
At this stage, you have acquired the basics in the area of traditional Linguistic semantics. We
will now take a look at the semantic phenomena usually called ‘figurative’ or ‘non-literal’, such as:
idiomatic expressions, metaphor, and metonymy.

• Idioms
Moon (1997, p. 43) states that idioms are a type of multi-word item. She defines a multi-
word item as an ‘item which consists of a sequence of two or more words. This sequence of
words semantically and/or syntactically forms a meaningful and inseparable unit.’ According to
Moon, an idiom must have a holistic meaning that cannot be inferred from the individual mean-
ing of the words that compose it, for example, have an axe to grind, or prepositional phrases, such
as over the top (Moon, 1997, p 46). Similarly, Cowie (1975, p. 8-9) defines an idiom as ‘a combina-
tion of two or more words which function as a unit of meaning’.
Example: I get on very well with her.
Meaning: We are good friends.

• Metaphor
Hurford, Heasley and Smith (2007, p. 331) explain that metaphors are conceptual (mental)
operations reflected in human language that enable speakers to structure and build abstract
areas of knowledge and experience in a more concrete experiential way. According to their view
of metaphor, speakers make use of a familiar area of knowledge, called the source domain, to
understand an area of knowledge that is less familiar, called the target domain. The source do-
main is typically understood through our experience about the physical world around us. See
the following examples about metaphorical expressions taken from Hurford, Heasley and Smith
(2007, p.330).
Example:
My car is a lemon.
Dr. Judith is a butcher.

In both examples complex and/or abstract areas of knowledge involving what we know
about cars and doctors have been emphasized in each metaphorical expression by linguis-
tically linking the more abstract target domains of knowledge about cars and doctors to more
particularized familiar concrete source domains (knowledge about lemons in the ‘fruit’ domain
and butchers in the domain of possible professions, respectively) in order to specify that there is
something negative about each (see more details in Hurford, Heasley and Smith (2007, p.331). As
(ibid.) acknowledge, we know from world general knowledge, for example, that lemons are sour
and that butchers can be messy and rough while doing their work. This knowledge enables us to
understand certain negative aspects about cars and the medical practice in an immediate way
via metaphor. Metaphors can be divided into three types:

1. Structural Metaphors
They are abstract metaphorical systems in which an entire (typically abstract) complex men-
tal concept is structured in terms of some other (usually more concrete) concept.
Example: Our ideas were right on target.

2. Orientational metaphor
This type of metaphor associates spatial orientation with an abstract knowledge area. The
examples show how human beings understand their orientation about physical space.
Example: Visiting her boosted my spirits.

35
UAB/Unimontes - 5º Período

Tip 3. Ontological metaphor


Idioms: An item con- One of the main uses of an ontological metaphor is to organize our understanding of ab-
sisting of more than stract concepts and experiences, such as events, activities, emotions, ideas, taking into consider-
one word forming a ation our experience with objects and substances in the real world.
meaningful unit; Example: We have to fight alcoholism for our own good.
Metaphors: Conceptual
operations reflected in
human language. When • Metonymy
using metaphors speak- As stated by Hurford, Heasley and Smith (2007, p. 338), metonymy is a kind of non-literal
ers construe an abstract language in which one entity is used to refer to another entity that is associated with it in some
concept; way. They put forward the idea that metonymic concepts enable us to conceptualize one thing
Metonymy: A kind of by means of its relation to something else.
non-literal language
in which one entity is Example: The ham sandwich in the next booth is waiting for his bill.
associated with a cor- As you can infer, we cannot interpret this example literally, as we know a sandwich will not
responding one. wait to get its bill. Such an interpretation would lead to an anomaly. The possible interpretation
for this sentence is that the person who ordered the ham sandwich is waiting for his bill. Another
point to take into consideration is the place where this sentence was uttered. Considering that
the sentence was uttered in a café or restaurant, the person uttering the sentence would know
Glossary that there was a close relationship between the thing ordered and the person who ordered it.
Expressões Because this relationship is easy to notice in the context, it is acceptable to refer to the person
idiomáticas (idioms): through what he has ordered.
são uma sequência de Another important characteristic of metonymies is that they can be subdivided into several
palavras ou frases que
semântica e/ou sintat- subtypes; check the following list.
icamente formam uma • THE PART FOR THE WHOLE
nova unidade de sen- • THE FACE FOR THE PERSON
tido reconhecido pelo • PRODUCER FOR PRODUCT
falante com base na cul- • OBJECT USED FOR USER
tura e no conhecimento
que o falante tem sobre • CONTROLLER FOR CONTROLLED
a língua. Ex: He kicked • INSTITUTION FOR PEOPLE RESPONSIBLE
the bucket (expression • THE PLACE FOR THE INSTITUTION
que significa morrer) • THE PLACE FOR THE EVENT
Metáfora: processo
cognitive pelo qual um
termo, pertencente a Check the example for ‘the institution for the people responsible’.
um determinado campo Example: The president’s office has announced his new agenda.
semântico é transferido We know for sure that people working closely to the president, for example, secretaries and
para outro. O campo assessors were the ones responsible for the announcement mentioned in the example given.
original é normalmente This section has observed different types of non-literal meaning. We hope you have found
referido como fonte
(source) e o novo como the discussion interesting and helpful. However, we have to emphasize that the issue of non-lit-
alvo (target). Ex: o termo eral and interpersonal meaning is vast and that it would be quite impossible to cover all the in-
cabeça (head) pode stances of both concepts. Nevertheless, we are sure that now you can expand your knowledge
ser usado no sentido on semantic meaning making connections to other important areas.
de parte de um corpo
vivo, como também no
sentido de principal. Ex:
He is the head of the de-
partment (Ele é o chefe
do departamento).
4.10 Summing Up This Unit
Metonímia: processo
pelo qual um elemento
é conceitualizado por We have outlined the characteristics of interpersonal and non-literal meaning. Before we
de outro referente. Ex: move on to the activities, you shall take a look at the reminders, they summarize the main topics
O sanduíche daquele covered in this unit. Make sure that you understand them. Do go back to the unit, if you have
mesa está aguardando doubts before doing the activities.
a conta. O termo
sanduíche foi utilizado
para referir-se o freguês.

References
AUSTIN, J. How to do Things With Words. Oxford: Claredon Press, 1962.

CHERCHIA, Genaro & McCONNEL, Sally. Meaning and grammar: an introduction to semantics.
Massachusetts: The MIT Press, 1995.

36
Letras Inglês - English Semantics

GRICE, H. P. Logic and Conversation. In. P. COLE: J. L. MORGAN. Syntax and Semantics 3: Speech
Acts. New York: Academic Press, p. 41-58, 1975.

HURFORD, James R. & HEASTLEY, Brendan. Semantics: a coursebook. Cambrige: 58 CUP, 1998.

HURFORD, HEASLEY and SMITH. Semantics: a course book. Cambridge: Cambridge University
Press, 2007

LEVINSON, S. Pragmatics. Cambridge: Cambridge University Press, 1983.

LEECH, Geoffrey. Semantics: the study of meaning. 2. ed. London: Penguim books, 1981.

SEARLE, J. Speech Acts: An Essay in the Philosophy of Language. Cambridge University Press,
1969.

YULE, G. Pragmatics. Oxford: Oxford University Press, 1996

37
Letras Inglês - English Semantics

Resumo
Unit 1

In this unit we went through the basic concepts of:


• Semantics
• Pragmatics
• Sentence, utterance and propositions
• Meaning in dictionaries
• Lexical semantics
• Word meaning and sentence meaning
• Sense relations (antonomy, homonomy, polysemy, synonomy, multi-meaning words and
ambiguity)
• Semantic field
• Language variation
Before we move on to the activities, you shall take a look at the reminders, they summarize
the main topics covered in this unit. Make sure that you understand them. Do go back to the
unit, if you have doubts before doing the activities.

Unit 2

In this unit, we went through these basic concepts:


• Reference
• Sense
• Anaphoric Reference
• Cataphoric Reference
• Exophoric Reference
Now, it is time to do the activities about Unit II to check your comprehension and practice it
a little more. Go ahead!

Unit 3

Dicionário: é parte central na descrição de qualquer língua. A entrada de um dicionário


apresenta, pelo menos, três informações básicas: (a) fonológica: indica como as palavras são
pronunciadas; (b) gramatical: apresenta a organização das palavras no discurso por meio de sua
função sintática; (c) semântica: apresenta informações sobre o significado das palavras em con-
texto de uso.

Propriedades do predicado - simetria X assimetria


A simetria ocorre quando os predicados têm um significado semelhante. Ex: João é casado
com Maria. Maria é casa com João.
A assimetria ocorre quando os predicados têm significados diferentes. Ex: Lucia é mais in-
teligente que Márcia (não se pode dizer que Márcia é mais inteligente que Lúcia).

Propriedades do predicado – reflexividade X irreflexividade


No fenômeno da reflexividade o significado do predicado referese integralmente ao ele-
mento a ele relacionado. Ex: Marta parece mais alta do que ela mesma. No fenômeno da irreflex-
ividade, o significado do predicado não está relacionado ao elemento a ele relacionado. Há,
nesse caso, um fenômeno de contradição. Ex: Paulo está diferente dele mesmo.
Propriedades do predicado – transitividade X intransitividade A transitividade ocorre
quando sentenças compostas são compatíveis umas com as outras. Ex: Mônica está no escritório
dela, que fica neste prédio. Nesse caso, ambos, Mônica e o escritório estão no mesmo local.
A intransitividade ocorre quando sentenças compostas são incompatíveis entre si. Ex:
Marta é mãe da Sofia. Nesse caso, não poderíamos dizer que Sofia seja mãe de Marta.
Derivação (derivation): é o fenômeno pelo qual ovas palavras podem ser foram, a partir de
um padrão regular, formado por palavras pré-existentes.

39
UAB/Unimontes - 5º Período

Morfemas (morphemes): são a menor unidade de análise morfológica.


Prefixo (prefix): são formas presas adicionadas ao início das palavras e que colaboram
para a formação do sentido. Ex: This is ungrammatical (un- indica negação, formando a palavra
não-gramatical ungrammatical).
Suffix (sufixo): são formas presas, adicionadas ao final das palavras, e que colaboram para a
formação do sentido. Ex: He is careless (less indica sem, faltando, formando a palavra descuidado-
careless)

Tipos de derivação
Inchoative: tipo de derivação indica o surgimento de um estado.
Ex: The room has enlightened with her presence.
Causative: tipo de derivação que indica que a uma determinada ação causou outra. Ex: The
strong wind closed the door.
Resultative: indica o resultado de uma ação. Ex: That window was broken by the ball.
Compound nouns: representam a combinação entre dois substantivos pré-existente que
forma um terceiro substantivo. Ex: pet-shop.

Participant roles (papéis dos participantes)


Agent (agente): pessoa ou coisa responsável pela ação representada pelo verbo. Ex: My
mother baked the cake. O agente é minha mãe (my mother), responsável pela ação de assar.
Affected (afetado): é a coisa ou pessoa afetada pela ação. Ex: The boys broje the window. O
ser afetado é janela (window), afetada pela ação de quebrar.
Instrumet (instrumento): é o elemento mencionado como meio pelo qual a ação é reali-
zada. Ex: They fond the place with a map. (mapmapa refere-se ao meio pelo qual o local foi encon-
trado.
Location (localização): é o elemento que indica o local onde uma ação foi realizada. Ex:
They visited London (Eles moram em Londres). In London (em Londres) indica local.
Beneficiary (beneficiário): indica o ser ou coisa que se beneficiou ou recebeu a ação ver-
bal. Ex: I gave him the post cards (o termo him-ele é o beneficiário da ação de dar).
Experience (experienciador): indica o ser que experiência a ação verbal. Ex: The girl listened
to the entire story. Nesse caso, a garota (girl) experienciou a ação de ouvir.
Theme (tema): é a coisa ou pessoa experienciada pelo experienciador na ação verbal. Ex:
The girl listened to the entire story.
Nesse caso, a estória (story) é o tema.

Unit 4

Speech acts (atos de fala): são atos sociais produzidos por meio da linguagem verbal. Ex:
pedir desculpas, saudar, reclamar, elogiar.
Atos de fala- categorias de análise Representativos: são atos de fala em que o falante
comprometesse com a verdade por ele expressa. Ex: This bag is blue (representa uma, asserção
sobre um elemento do grupo).
Diretivos: são atos de fala em que o falante compele o outro a fazer algo por ele. Ex: Come
to my room tomorrow.
Comissivos: são atos de fala pelos quais o falante compromete-se em realizar algo no
futuro. Ex: I Will come tomorrow.
Expressivos: são atos de fala pelos quais o falante expressa um estado psicológico de agra-
decimento, elogio, boas-vindas. Ex: Congratulations! You did a good job.
Declarativos (ou performativo): são atos de fala pelos quais o uso da linguagem modifica
a realidade. Ex: I declare you husband and wife.
Expressões idiomáticas (idioms): são uma sequência de palavras ou frases que semântica
e/ou sintaticamente formam uma nova unidade de sentido reconhecido pelo falante com base
na cultura e no conhecimento que o falante tem sobre a língua. Ex: He kicked the bucket (expres-
sion que significa morrer)
Metáfora: processo cognitive pelo qual um termo, pertencente a um determinado cam-
po semântico é transferido para outro. O campo original é normalmente referido como fonte
(source) e o novo como alvo (target). Ex: o termo cabeça (head) pode ser usado no sentido de
parte de um corpo vivo, como também no sentido de principal. Ex: He is the head of the depart-
ment (Ele é o chefe do departamento).

40
Letras Inglês - English Semantics

Metonímia: processo pelo qual um elemento é conceitualizado por de outro referente. Ex:
O sanduíche daquela mesa está aguardando a conta. O termo sanduíche foi utilizado para referir-se
ao freguês.

41
Letras Inglês - English Semantics

Referências
Básicas

CHERCHIA, Genaro & McCONNEL, Sally. Meaning and grammar: an introduction to semantics.
Massachusetts: The MIT Press, 1995.

HURFORD, James R. & HEASTLEY, Brendan. Semantics: a coursebook. Cambrige: 58 CUP, 1998.

LEECH, Geoffrey. Semantics: the study of meaning. 2. ed. London: Penguim books, 1981.

Complementares

AIJMER, K. Conversational routines in English: Convention and Creativity. New York: Longman,
1996.

AUSTIN, J. How to Do Things with Words. Oxford: Clarendon Press, 1962.

CARTER, R.A., and M.J. McCARTHY. The Cambridge Grammar of English: spoken and written
English Grammar and Usage. Cambridge: Cambridge University Press, 2006.

GRICE, H. P. Logic and Conversation. In. P. COLE: J. L. MORGAN. (eds.) Syntax and Semantics 3:
Speech Acts. New York: Academic Press, p. 41- 58, 1975.

HURFORD, HEASLEY and SMITH. Semantics: a course book. Cambridge: Cambridge University
Press, 2007.

LEECH, G. Principles of pragmatics. London: Longman, 1983

LEVINSON, S. Pragmatics. Cambridge: Cambridge University Press, 1983.

McCARTHY, M.J. Applied Linguistics. Cambridge: Cambridge University Press, 1998.

OLIVEIRA, NÁDIA, A. de. Para ler em inglês: desenvolvimento da estratégia de leitura. Belo Hori-
zonte: O Lutador, 1988.

SEARLE, J. 1969. Speech Acts: An Essay in the Philosophy of Language. Cambridge: Cambridge
University Press.

YULE, G. Pragmatics. Oxford: Oxford University Press, 1996.

43
Letras Inglês - English Semantics

Atividades de
Aprendizagem - AA
1) Could the following possibly be used as referring expressions?
Circle the answer of your choice.

1. Mary Yes / No
2. His mother Yes / No
3. However Yes / No
4. The new English teacher Yes / No
5. A baby Yes / No
6. My keys Yes / No
7. Receive Yes / No
8. Above Yes / No

2) Which of the following statements are SENTENCES? Indicate your answer by circling Yes or No.

1. Do all (authentic) performances of Macbeth begin by using the same sentence? Yes / No
2. Do all (authentic) performances of Macbeth begin with the same utterance? Yes / No
3. Does it make sense to talk of the time and place of a sentence? Yes / No
4. Does it make sense to talk of the time and place of an utterance? Yes / No
5. Can one talk of a loud sentence? Yes / No
6. Can one talk of a slow utterance? Yes / No

3) Mark each of the following statements true (T) or false (F).

1. Alive means the opposite of dead. T / F


2. Buy has an opposite meaning from sell. T / F
3. Caesar is and is not a meaningful English sentence. T / F
4. Caesar is a prime number is nonsensical. T / F
5. Caesar is a man is nonsensical. T / F
6. Both of John’s parents are married to aunts of mine is in a sense contradictory, describing an
impossible situation. T / F
7. If the sentence John killed Bill is true of any situation, then so is the sentence Bill is alive. T / F
8. If someone says, ‘Can you pass the salt?’, he is normally not asking about his hearer’s ability to
pass the salt, but requesting the hearer to pass the salt. T / F
9. If someone says, ‘I tried to buy some rice’, his hearer would normally infer that he had actually
failed to buy rice. T / F

4) What is the semantic relationship between (or among) the following words: synonymy, an-
tonymy, polyssemy, hyponymy. The first ones are examples:

couch - sofa SYNONYMY


awake- asleep ANTONYMY
take: grasp,carry POLYSEMY
stand (be on my feet) – stand (resist) HOMONYNY

a) stop- go ________________
b) strong /weak ________________
c) do- undo ________________
d) fast- slow ________________
e) mammal- dog, cat, pig, cow________________
f ) alive- dead ________________
g) get - obtain, become, buy ________________

45
UAB/Unimontes - 5º Período

h) beginning - debut ________________


i) It is their book - They live there ________________

5) Remember: HOMOPHONES are words that sound the same and are spelled differently and
have different meanings.
Read the sentences below and fill in the gaps with the appropriate homophone.
Go to www.dictionary.com, if you need any help!

1) Honey comes from the ________ .


• Be
• Bee

2) I would like a table ______ _________ , please.


• Four
• fore
• for

3) After feeling ill, she got very _______


• Pail
• pale

4) I don´t know ________ he is coming or not.


• Weather
• whether

5) I never come ________ because I can´t ___________ their voice!


• Hear
• here

6) There is ________ room available. Do you __________ where the nearest hotel is?
• Know
• no

7) She must ___________ the letter ___________now.


• Write
• right

6) Go to www.dictionary.com and find out the meaning of the words in bold. Consider the differ-
ent linguistic contexts in which they appear. The first one is done for you:

Arm I have a pain in my arm. (part of the body)


(noun)
Policemen in Japan do not carry any arms.(weapons)

Ask You should ask your teacher.


(verb)
May I ask you a favor?

Bachelor As a wealthy bachelor, he should have no problem finding


(noun) someone to marry him.
It normally takes 4 years to get a bachelor’s degree.

Ball A soccer ball is 28 inches in circumference.


(verb)
Everyone had a good time at the ball.

46
Letras Inglês - English Semantics

Bank He works for a bank.


(noun)
Flooding has caused repeated damage to the river bank.

Can Can I drive?


(verb/noun)
Could I have a can of soda please?

Character Running away from difficulties was not part of his character.
(noun)
Characters are an essential element of fictional works,
especially novels and plays.

Gas The price of gas is going up as a result of the instability in the


(noun) Middle East.
Oxygen is the gas that supports life.

Letter I got a letter from my friend.


(noun)
“A” is the first letter of the alphabet.

Party I’ve been invited to a birthday party.


(noun)
I’m going to join the liberal party.

7) Remember: synonyms are words which have similar meanings. Find out the synonyms for the
words given. Number them correctly. If you need any help, try: www.dictionary.com.br

1) polite ( ) silly
2) temper ( ) mood
3) rude ( ) impolite
4) selection ( ) poisoned
5) foolish ( ) choice
6) toxic ( ) well- mannered
7) meeting ( ) infantile
8) childish ( ) assembly

8) Identify the right speech act in the following sentences.

a) I am so sorry I didn’t mean to hurt you.


b) I declare you husband and wife.
c) The boss suggested it would be better to change the strategy.
d) Do you ant water or any other thing to drink?
e) Congratulations! You did very well.

9) Match the descriptions with the sentences. You might need to go back to metonymy.

1. The EU is disappointed with the poverty rates of ( ) Producer for product.


some countries. ( ) Institution for people responsible.
2. Roads were busy this morning. ( ) Place for the event.
3. You need ´leite moça´for this recipe. ( ) Thing used for user.
4. This house goes well with parties.

47
UAB/Unimontes - 5º Período

10) Match the words in the box to the sentences with the corresponding meaning.
Busy, start, a lot of, kind, sad.

• Will you be a peach and bring that chair to me?__________


• I am up on the walls with so much work.___________
• Let’s get the ball rolling and finish the work soon. ___________
• I feel very low these days.__________
• There is a sea of people in front of me.___________

48

Você também pode gostar